SlideShare una empresa de Scribd logo
1 de 61
GMAT VERBAL REVIEW
SENTENCE CORRECTION
SENTENCE STRUCTURE
PHRASE CLAUSE SENTENCE
A clause is a group of related words that
must have a subject and a verb. Clauses
may be classified into two categories:
• An independent clause is one which
contains a subject and a verb, and
expresses a complete thought. For
example, Jane took the train
• A dependent clause is one which
contains a subject and a verb, but does
not express a complete thought, and
cannot stand for itself. For
example, after Jane took the train. The
clause raises a question – what
happened after Jane took the train?
Example: Barry ran, as he wanted to reach
on time
Independent clause: Barry ran
Dependent clause: as he wanted to reach
on time
Group of related words which does not
express a complete thought, and where
the subject, verb or both, are missing
• Prepositional phrase: at the beach,
near the stove, at this time
• Verbal Phase: building a large dam, to
fly in a plane, signed the legal
document
Examples:
- The sun rises in the east
- Humpty Dumpty sat on a wall
- She wore a hat with blue trimming
- The accident on the bridge was not
serious
- The girl with red hair is an artist
• Must have a subject
• Must have a verb
• Must make common-sense/form a
complete thought
COMMON ERRORS
Common splice
This occurs when two independent
clauses in a sentence are combined using
a comma instead of using a semi-colon or
an appropriate conjunction
Incorrect: Tom read the novel, his friend
saw the movie
Correct: Tom read the novel, but his friend
saw the movie
Correct: Tom read the novel; his friend
saw the movie
PARTS OF SPEECH
Part of speech Definition Example
Noun Concrete nouns: person, animal, thing, place
Abstract nouns: You cannot see them, hear them, smell them, taste them, or feel them
(physically)
Concrete nouns: teacher, airplane, dog
Abstract nouns: curiosity, love, happiness
Pronoun Word that takes the place of a noun
Example: I, you, he, she, it, we, them, me, you, him, her, it, us, them, mine, yours, his,
hers, its, ours, theirs, this, that, which, who
Verb Verbs express action – something that a person, animal, force of nature, or thing can
DO. For instance, sneezing, laughing, winking
State of being – linking verbs connect the subject of a verb to additional information
about the subject. Some examples include, any form of the verb be – am, were, has
been, are being, might have been – become, and seem
A three-mile run seems like a marathon
during a hot, humid July afternoon
During bad storms, trailer parks are often
magnets for tornadoes
Adjective A word that modifies a NOUN or a PRONOUN by describing, identifying, or quantifying
words
- What kind?
- Which one?
- How much?
Smith is a famous explorer
Sidney Sheldon has written thirty novels
Adverb A word that modifies everything but nouns and pronouns – modifies ADJECTIVES,
VERBS AND OTHER ADVERBS. Most adverbs end in –ly.
A word is an adverb if it answers
- How?
- When?
- Where?
- Why?
We’ll use the new software program
tomorrow
He climbed the ladder slowly
Conjunction Connecting words: and, but, or, nor, for, yet, so, although, because, since, unless Jack and Jill went up the hill
Preposition Links nouns, phrases and pronouns to other words in the sentence The book is on the table
THREE KEY CONCEPTS IN SENTENCE CORRECTION
CONCISION
Refers to brevity –
is the sentence
written as
economically as
possible?
Avoid redundancy - if a word can be removed without altering the meaning of the sentence, it is redundant and should be eliminated
Example:
• Past experience reveals that cancer patients rarely ever exhibit the exact same symptoms
• In the example above, the word “past” is implicit in “experience” and can be eliminated. Similarly the words “ever” and “exact” are
only used for emphasis (a practice the GMAT does not condone)
• Correct: Experience reveals that cancer patients rarely exhibit the same symptoms
A common redundancy trap is the use of words with the same meaning
Example:
• The value of the stock rose by a 10% increase
• Since rose and increase both imply growth, only one is needed to convey the correct meaning
“Being” almost always signals redundancy and should be avoided wherever possible
CORRECTNESS Subject – verb agreement; Verb tense; Voice & mood; Pronouns; Modifiers; Parallelism; Comparisons; Idioms
CLARITY
Refers to
intelligibility – is
the meaning of the
sentence obvious
and unambiguous?
If an answer choice alters the original meaning (or intent) of the sentence, it is incorrect. Most instances of altered meaning fall into four
broad categories:
1. Word placement
2. Known vs. Unknown
i. The original sentence is certain about an outcome but the answer choices indicate uncertainty (or vice versa)
ii. The original sentence discusses a hypothetical situation but the answer choices present it as an actual situation (or vice versa)
3. Multiple meanings
4. Such as vs. Like
THREE KEY CONCEPTS IN SENTENCE CORRECTION – CLARITY CONTINUED
Category Description Example
Word Placement Be aware of words that move from one position
to the other; the placement of a single word can
alter the meaning of the sentence
All the children are covered with mud
The children are all covered with mud
In these sentences, the placement of all shifts the focus from the number of
children covered with mud to the extent to which the children are covered
by it
Known vs. Unknown The original sentence is certain about an
outcome but the answer choices indicate
uncertainty (or vice versa)
Certain: The court ruled that the plaintiff must pay full damages
Uncertain: The court ruled that the plaintiff should pay full damages
The original sentence discusses a hypothetical
situation but the answer choices present it as an
actual situation (or vice versa)
Actual: If Newton and Liebniz met, they discussed mathematics
Hypothetical: If Newton and Liebniz met, they would discuss mathematics
Multiple meanings Some words can be read in more than one way,
altering the meaning of the sentence according
to the given interpretation
Sentence # 1: The light fabric makes the shirt easy to fold
Sentence # 2: The shirt is easy to fold, and very light
The first sentence makes it clear that in this context the shirt is “not heavy”.
However, in the second sentence, light could mean either “pale” or “not
heavy”. The context does not establish the meaning clearly
“Such as” vs. “Like” “Such as” is used to indicate examples; “Like” is
used to indicate similarity
Animals such as zebras and lions live on the Serengeti Plain
Animals like zebras and lions live on the Serengeti Plain
The first sentence indicates that lions and zebras are specific types of
animals that live on the Serengeti Plain. The second sentence indicates that
animals similar to lions and zebras live on the Serengeti Plain, but it is not
clear whether lions and zebras themselves actually do
SUBJECT – VERB AGREEMENT
The basic rule states that a singular subject takes a singular verb, while a plural subject takes a plural verb
First determine whether the subject in the sentence or the clause is singular or plural
• To find the subject, always find the verb first, and then ask who or what performed the verb
• It is important to note that verbs do not form their plurals by adding an s as nouns do. In order to determine which verb is singular and which one is
plural, think of which verb you would use with he or she and which verb you would use with they
• If a verb follows to, it is called an infinitive phrase and is not the main verb. You will find the main verb either before or after the infinitive phrase
Example: The efforts to get her elected succeeded
ELIMINATE THE MIDDLEMAN
The intervening phase between the subject and the verb should be
eliminated so that the true subject becomes clear
• “of” and “for” are two common middlemen
• Prepositional phrases
Examples:
The houses of that rich man (contain/contains) very expensive furniture
The subject – houses – is plural, and hence requires the plural verb form
contain
• The discovery of new lands (was/were) important to the expansion of
the British empire
• The building of tall skyscrapers (has/have) increased in the past few
years
• The actions of my friend (is/are) not very wise
• The book I bought for my students (tell/tells) the story of a Russian
immigrant’s experience in the United States
COLLECTIVE NOUNS ARE GENERALLY SINGULAR
A collective noun is a noun that looks singular (it usually does not end with
an “s”) but refers to a group of people
Collective nouns are singular when they act in a collective fashion or
represent a group. Examples include
team, family, majority, administration, army, audience, class, crowd, facult
y, audience, and committee
Collective nouns are plural when the members of the collective body act
as individuals
• The team are always fighting amongst themselves
• While 'team' is often used as a singular collective noun, in this case, the
sentence describes the fighting that occurs between the individual
members of the team. "Team" therefore refers to several individual
members, and requires a plural verb, "are," as a result
SUBJECT – VERB AGREEMENT
AND vs. ADDITIVES
The word “and” can unite two or more singular subjects, forming a compound
plural subject
• John and his friends ARE going to the beach
When the two singular noun subjects connected by “and” refer to the same
person or thing, the singular verb is used
• Bread and butter is his staple diet
• The recently appointed manager and coach of the team has emphasized
the need for fitness
Additive phrases such as: along with, in addition to, as well as, accompanied by,
together with, including, do not form compound plural subjects
• Correct: John, along with his friends, IS going to the beach
• Incorrect: Frank, accompanied by his students, were at the studio
• Correct: Frank, accompanied by his students, was at the studio
INFINITE PRONOUNS ARE SINGULAR
An infinite pronoun is one which is not specific about the thing to which it refers
All pronouns which end in –body, – one, or –thing fall under this category:
- Anyone, anything, anybody
- Everyone, everybody, everything
- Each, every
- Whatever, whoever, no one, nobody, nothing
- Either and neither (may require a plural verb form when paired with or/nor)
• Neither of the rosebushes is as pretty as last year
• Either of us is capable of doing the work
• Neither John nor his friends are going to the beach
There are 5 infinite pronouns which can be either singular or plural depending on
the context of the sentence: Some, Any, None, All, Most (SANAM)
VERB FORM IN EITHER/OR , NEITHER/NOR DEPENDS ON
THE SUBJECT NEAREST TO THE VERB
In these sentences, there are two subjects. If one subject is singular, and the other
is plural, find the subject nearest to the verb, and ensure that the verb agrees in
number with this subject
• Neither Joe nor his friends ARE going to the beach
• Neither his friends nor Joe IS going to the beach
EACH/EVERY – GENERALLY SINGULAR SENSATIONS
When each/every is the subject, the verb form is singular
• Each of these shirts IS pretty
When the subject is preceded by each/every, the verb form is singular
• Every dog and cat HAS paws
When each/every follows a subject, it has no bearing on the verb form
• They each ARE great tennis players
• The apartments each HAVE their own private entrance
SUBJECT – VERB AGREEMENT
NUMERICAL
WORDS AND
PHRASES
The phrase, “the number of” always takes a singular verb form
The phrase, “a number of” always takes a plural verb form
Numbers greater than 1 are plural
• The number of students in this class IS quite large
• A number of students in this class ARE hardworking
• TWO out of every three dog owners in the U.S. also OWN a cat
PORTIONS With words that indicate portions—percent, fraction, part, majority,
some, all, none, remainder, and so forth —look at the noun in your of
phrase (object of the preposition) to determine whether to use a
singular or plural verb
If one means the many individual parts of the totality, then use a plural
verb form
Example:
The majority of the students in the class ARE hard working
If one means the totality itself, then use a singular verb form
Example:
The student majority IS opposed to the death penalty
• Half of the dresses ARE dirty
• Half of the sugar IS over
• Two-thirds of the work IS complete
• The majority of Congress has voted for the bill
“ONE OF” The phrases one of and one of the take a singular verb • One of you is telling the truth
• One of the editors wants a rewrite
Plural - The phrases one of those who and one of the things/factors
Singular - When the word “the only” precedes these phrases
• The comma splice is one of those errors that always slip past me
• One of the things that drive me nuts is subject-verb agreement
• Meg is the only one who knows how to paddle a canoe
• Ted is not the only one of my nephews who has a vivid
imagination
MISCELLANEOUS
• “Both... and” is the ONLY pair that always results in a plural subject
• Gerunds (-ing form of the verb and functioning as nouns) are singular subjects
• When in doubt, think singular
• When a sentence has both a positive and a negative subject, the verb agrees with the positive subject. For example: Low prices and not quality
determine many purchases; Peace of mind not riches is what makes a person happy
VERB TENSE
VERB TENSE indicates when an action takes place. The basic tenses are present, past and future
Unless actions do not take place at the same time, keep all verb tenses in a given sentence the same
Example
• She walked to school in the morning and ran home in the afternoon
• She walks to school in the morning and runs home in the afternoon
• She will walk to school in the morning and run home in the afternoon (Note: run is understood as will run)
• Typically, –ing forms are sometimes used as junk answers on the GMAT; there will often be a better alternative
• Tip: Pick one event as the base action  determine when the other actions occurred in relation to that event  determine the proper verb form
• “Will” indicates future tense; “would” indicates future from the past
• “Might” is the past tense of “May”
• Progressive tense indicates an ongoing action – in the past, present or future
• The perfect progressive form indicates an action that is ongoing, but will be completed at some definite time
• Infinitive verbs: “To” + verb. This is considered as the most basic form of the verb, or the building block of all other tenses. To form other tenses of the verb, you
simply modify the infinitive form
- Avoid sentences that insert a word (or words) between “to” and the verb (rarely tested, but almost always incorrect)
IF & THEN CLAUSE
IF Clause THEN clause Example
Present will + base verb If she wins the lottery, she will give half the money to charity
Past would/could + base verb If she won the lottery, she would give half the money to charity
Future would/could + have + past participle If she had won the lottery, she would have given half the money to charity
** Conditional words “would” and “could” NEVER appear in the IF clause
VERB TENSE - PRESENT TENSE
Tense When to Use How to Use
Simple present Generally expresses events, or situations that have existed in the past, exist now (in the present) and will probably
exist in the future
- I understand what you are saying
General truths
- Grass is green
Present perfect If an event started in the past but continues (or remains true in) in the present / actions have not yet finished
- We have lived in a little hut for three days
- Our country has enforced strict immigration laws for thirty years
When an event occurred at an indeterminate point in the past
- They have known each other for the longest time
- I have been to California
If the time period has not finished
- I have seen three movies this week
Giving recent news
- Michael has crashed his car again
Has/Have + Past participle. Past participle
of a regular verb is verb + -ed ending
(example: walked, danced, jumped)
Present perfect
continuous
Recent activity
- She's been working hard recently
Emphasis on the duration or length of an activity
- Jack has been painting for 4 hours
Recently finished activity with a present result
- I've been working in the garden, that's why my hands are so dirty
Present continuous If an event is occurring at this very moment / around now / near future (especially planned activities)
- The kids are watching TV
- We aren't working hard these days
- Polly is coming for dinner tomorrow
Am/is/are + verb-ing
VERB TENSE - PAST TENSE
Tense When to Use How to Use
Simple past If an event started and finished at a particular point of time in the past
- Molly and Rita embraced at the airport
- George ate all his dinner
Past perfect If more than one event in the sentence occurred at different times in the past, use the past perfect tense for the
earlier action (FINISHED action) and simple past for the later action
- The film had started by the time we arrived at the theatre
- The teacher thought that Johnny had cheated on his exam
Had + past participle
Past perfect
continuous
Past perfect progressive is used to express CONTINUOUS activity up to a specific point of time in the past
- They had been waiting for 2 hours before their friends finally arrived
Past continuous Duration in the past - actions or situations that lasted for some time in the past, and whose duration time is
unknown or unimportant
- The dog was barking
Interrupted actions in progress – when one action in progress is interrupted by another action in the past. We
usually use when or while to link these two sentences.
- I was talking with James when the telephone rang
Actions in progress at the same time in the past - two or more activities happening at the same. We usually use
when or while to link the two sentences
- When Bob was painting windows, Mary was working in the kitchen
VERB TENSE - FUTURE TENSE
Tense When to Use How to Use
Simple future Promises
- I promise I will buy you this toy
Unplanned actions (spontaneous decisions)
- Don't worry! I will help you with this problem
Predictions based on experience or intuition
- It will rain in a moment
Habits (obstinate insistence, usually habitual)
- She will bite her lip if she is thinking or if she's nervous about something
** Remember, you should never use will to say what somebody has already arranged or decided to do in the
future:
Correct: Mike is moving to New York next week
Incorrect: Mike will move to New York next week
Future perfect The Future Perfect expresses the idea that something will occur before another action in the future. It can also
show that something will happen before a specific time in the future
- I will have been in London for six months by the time I leave
- Sam is probably going to have completed the proposal by the time he leaves this afternoon
Incorrect: By the time I write to Leo, he will probably move
Correct: By the time I write to Leo, he will probably have moved
Will have + past participle
Am/is/are + going to have + past
participle
Future perfect
continuous
Emphasize the length of time or duration of an event occurring before and up to another event in the future
- John will have been studying for 6 years by the time he finishes his exam
Future continuous Future actions in progress
- In an hour, I will be sitting in front of my TV
Guesses about the present/future
- He won't be coming any time soon. He is still at the office
Polite questions about somebody's intention
- Will you be going to the supermarket? I have something to buy
Will + be + verb-ing
MOODS
SUBJUNCTIVE
Verbs in subjunctive mood deal with events that are not necessarily true
INDICATIVE
Verbs in indicative mood deal with real events
IF CLAUSE
When the “if” clause represents
something contrary to reality - In
these cases, the verb is always
“were”, regardless of the subject
• If he were tall, he would be able to
play basketball better
“That” and “That” Parallelism is to be
maintained
• Incorrect: He said that he had
studied well and he would score
well
• Correct: He said that he had
studied well and that he would
score well
Hopes, dreams, desires and requests formed with
the word “that”
It is the desire of one person or body for another
person or body to do something. It is unclear as to
whether or not the second person or body will
actually do what is asked
Subjunctive is formed with “that + base form of the
verb” and follows words such as advice, advisable,
ask, arrange, better, demand, desire, instruct,
instructions, intend, intentions, order, pray, prefer,
propose, recommend, suggest, request, require,
urge, urgent, vital etc. The base form of the verb is
always plural
• Correct: The parolee knew it was imperative that
he find a job quickly (base form of the verb)
• Incorrect: It is imperative that she must sign the
permission slip (must is redundant)
• Correct: It is imperative that she sign the
permission slip
VOICE
ACTIVE VOICE PASSIVE VOICE
The subject of the sentence has an action performed
on it by someone or something else
- The pizza was eaten by the hungry students
Passive voice is formed with a form of the verb “to be”,
followed by a participle. The person or people
performing the action in the sentence almost always
follow the verb
Required when the non-underlined portion of the
sentence contains the person or agent performing the
action preceded by the word “by”
• Incorrect: The shuttle launch seen around the
world by people of all ages, all races and all
religions (missing a verb, and therefore is a
fragment)
• Correct: The shuttle launch was seen around the
world by people of all ages, all races and all
religions
SUBJECT PERFORMS THE ACTION
- The hungry students ate the pizza
Active voice is preferred to passive voice. Only transitive verbs (verbs that take direct objects) can be written in passive voice
- The aliens arrived on Neptune in the 20th century
PRONOUNS
Subject Pronouns Object Pronouns Possessive Pronouns
I Me My, mine
You You Your, yours
He Him His
She Her Her, hers
It It Its (not it’s)
We Us Our, ours
They Them Their, theirs
Who Whom Whose
Who vs. whom
To simplify who/whom questions, try rearranging the sentence
into a question, and then answer it
Question: Who/m did Kate marry?
Answer: Kate married him
Since the pronoun used in the answer is "him," an object pronoun,
the pronoun in the original sentence should also be an object
pronoun: whom
Example # 1
• Incorrect: Janice and ME went on a picnic together
• Correct: Janice and I went on a picnic together
• The pronoun is part of the subject, and hence, the correct form is I and not me
Example # 2
• Incorrect: The picnic was attended by Janice and I
• Correct: The picnic was attended by Janice and me
• Picnic is the subject, while the pronoun is part of the object, thus the correct
form is me and not I
Example # 3
Marston was an early seventeenth century dramatist and it is likely that him and
Shakespeare borrowed ideas from one another.
• It is likely that him and Shakespeare borrowed ideas from one another
• It is likely that they borrowed ideas from one another
• It is likely that him and Shakespeare borrowed ideas from each other
• It is likely that himself and Shakespeare borrowed ideas from one another
• It is likely that he and Shakespeare borrowed ideas from one another
Explanation: This question tests pronoun agreement. The pronoun ‘him’ in the
original sentence replaces ‘Marston.’ It is in the wrong case. Instead of the objective
case, the pronoun should be in the subjective case, since Marston is the subject of
the sentence. Therefore (E), which uses the subjective pronoun ‘he,’ is correct. (B) is
wrong because, though ‘they’ is subjective, it eliminates any meaning of
Shakespeare from the sentence, making the pronoun ambiguous
• Adding a pronoun in an answer choice more often than not
makes the answer choice wrong. However, sometimes it is
essential to introduce a pronoun to add clarity
• The first step to solving a pronoun question is to identify all the
pronouns
• A relative pronoun (i.e., that, which, who etc.) must refer to the
word immediately preceding it
• A single pronoun can only refer to a noun, not a noun and a verb
PRONOUNS – 1
Implication is not enough; there must be a stated antecedent for
every pronoun
Friendship was something that James always valued, so he
disliked it when THEY talked about him behind his back
He, him and his clearly refers to James. However, the pronoun
THEY has no antecedent at all. One might think that THEY refers
to friends, but the word friends is never mentioned in the
sentence, only the word friendship is. Therefore, the sentence is
incorrect
A pronoun must refer to one and ONLY one antecedent (noun)
After finding the antecedent, check whether the pronoun agrees
in number
** The deadly four: it, its, they, their
• Incorrect: Police work is very important as THEY help to
enforce the laws of the state
• Correct: Policemen are very important as THEY help to
enforce the laws of the state
• Correct: Police work is very important as IT is the backbone of
the state
Example # 1
• Incorrect: When the person calls, take down THEIR phone
number
• Correct: When the person calls, take down HIS phone number
• Correct: When the people call, take down THEIR phone
number
Example # 2
• Incorrect: Everyone here will need THEIR own pencil
• Correct: Everyone here will need HIS own pencil
Pronoun must agree in number (singular/plural) with the
antecedent
PRONOUNS – 2
Possessive pronouns can refer back to possessive nouns.
However, subject and object pronouns may NOT refer back to
possessive nouns. Subject and object pronouns may only refer
back to subject and object nouns
• Incorrect: Jose’s room is so messy that his mother calls HIM a
pig
• Correct: Jose’s room is so messy that his mother calls Jose a
pig
POSSESSIVE POISON
Never use one or one’s to refer back to any antecedent except
one. Similar for you
Example # 1
• Incorrect: A person should leave a light on in an empty house
if one wants to give the impression that someone is at home
• Correct: Incorrect: A person should leave a light on in an
empty house if he or she wants to give the impression that
someone is at home
• Correct: One should leave a light on in an empty house if one
wants to give the impression that someone is at home
Example # 2
• Incorrect: One should have their teeth checked every six
months
• Correct: One should have one's teeth checked every six
months
• Correct: You should have your teeth checked every six
months
IMPERSONAL PRONOUNS – ONE / YOU
MODIFIERS
A modifier or a modifying phrase describes someone or something in the sentence
A modifier must always be placed as close as possible to the word it's modifying
ADJECTIVES
An adjective describes ONLY a noun or pronoun and
answers the questions: how many, which one, what kind?
The following “sense” verbs (verbs that describe
someone's sensation or feeling or perception) require
adjective modifiers:
- Sound
- Look
- Smell
- Taste
- Feel
- Seem
• The smart man acts quickly
• The adjective smart modifies the noun man, while the
adverb quickly modifies the verb acts
• Incorrect: After she returned from the three-week
vacation, she looked very well (looked is a sense word,
well is a adverb)
• Correct: After she returned from the three-week
vacation, she looked very good (adjective)
ADVERBS
An adverb usually modifies a verb, but it can also describe
an adjective, another adverb, a preposition or a phrase.
Many adverbs are formed by adding –ly to the adjective
• Incorrect: My friend is a real interesting person
• Correct: My friend is a really interesting person
• The adverb really must be used to modify the adjective
interesting
MODIFIERS – 1
MODIFIER ERRORS
POSSESSIVE POSIONDANGLING MODIFIERS MISPLACED MODIFIERS
When the modified noun is not present
• Incorrect: Using the latest technology, the
mechanical problem was identified quickly
• The modifier, using the latest technology, is
probably describing the technician who
identified the problem. However, a
technician never appears in the sentence
• Correct: Using the latest technology, the
engineer identified the problem quickly
This occurs when the modified noun is not placed directly
next to the modifying phrase.
A modifying phrase must not be separated from the noun it
modifies
A modifying phrase that begins a sentence refers to the
noun or pronoun immediately following the phrase
Example # 1
• Incorrect: Upon leaving the register, the cashier handed
the customer a receipt
• The modifier, upon leaving the register seems to modify
the cashier, although it should modify the customer. In
order to correct this, we must place the modifying
phrase directly next to what it modifies
• Correct: Upon leaving the register, the customer
received a receipt from the cashier
Example # 2
• Incorrect: Kendra is happy, like all her friends, to be on
vacation
• Correct: Kendra, like all her friends, is happy to be on
vacation
• Incorrect: Unskilled in complex math,
Bill’s score on the entrance exam was
poor
• Correct: Unskilled in complex math, Bill
did not score well on the entrance exam
MODIFIERS – 2
RELATIVE PRONOUNS - which, that, where, who, whose, whom
Relative pronouns introduce relative clauses, which are a type of dependent clause. Relative clauses modify a word, phrase, or idea in the main clause
Who introduces phrases that modify a person or a group of people (if the antecedent is in the object, then whom is to be used)
Which introduces phrases that modify things
That can be used to modify either people or things
Where can be used to modify to places
In which = places/ situations/time period/ phenomenon
Example
Incorrect: The Yankees, never liking to lose, practice everyday
Correct: The Yankees, who never like to lose, practice everyday
Correct: One of the books, which were on the table is mine
• That is preferred to which
• That is typically used to refer to singular nouns, and those/these is used to refer to plural nouns
• ,which ,that ,who ,whom ,it, whose ,it - refers to the immediately preceding noun, and not to the action of the entire preceding clause
Example
• The police found the murder weapon, which made the prosecutor’s job much easier
• The above sentence literally means that the murder weapon itself rather than finding the murder weapon made the prosecutor’s job easier
• Better: The police found the murder weapon making the prosecutor’s job much easier
MODIFIERS – 3
ESSENTIAL VS. NON-ESSENTIAL MODIFIERS
“Which” is used to introduce non-essential modifiers – can be removed from the sentence without the sentence losing any essential meaning
“That” is used to introduce essential modifiers – essential to the meaning of the sentence
Commas are used to separate non-essential modifies from the noun that is modified
• Non-essential: To find my house, walk down the left side of the street until you reach the third house, which is red
• Essential: To find my house, walk down the left side of the street until you reach the third house that is red
• Non-essential: This is my uncle John, who lives in Toronto
• Essential: This is my uncle John who lives in Toronto
• Only guests who are accompanied by tenants may use the pool
• Only guests, who are accompanied by tenants, may use the pool
• The first sentence identifies a specific group of guests who may use the pool: those accompanied by tenants (as opposed to those
unaccompanied by tenants). By contrast, the second sentence indicates (somewhat illogically) that only guests (as opposed to tenants) may
use the pool, and that they just happen to be accompanied by tenants. The second sentence contains a non-essential clause where an
essential clause is required
AND/BUT/ALTHOUGH
When there is an and/but/although in a sentence, then the pronoun refers to the first noun or the main subject
• Although Mary is younger than Susan, she is more mature
PARALLELISM – 1
PARALLEL STRUCTURES
Nouns Trevor collects stamps, coins and cards
Adjectives The wait staff was prompt, friendly and competent
Modified nouns A positive attitude can lead to both practical success and spiritual fulfillment
Verbs We worked all day, ate all evening, and slept all night
Verb infinitives I decided to swim across the river than sail around the world (the second to is optional)
Participial phrases The rain continued to fall, providing water for thirsty plants but flooding the streets as well
Adverbs I’ve noticed that you often howl angrily after you cower fearfully
Adverbial phrases I’ve noticed that you often howl in anger after you cower in fear
If one item includes a pronoun, it is often
appropriate to include the same pronoun in
all items
• Incorrect: I prefer to hire employees who
work hard to those that don’t
• Correct: I prefer to hire employees who
work hard to those who don’t
Only structures that are logically parallel
must be structurally parallel
• Correct: Ken traveled around the world,
visiting historic sites, eating native foods,
and learning about new cultures
• Incorrect: Ken traveled around the world,
visited historic sites, ate native foods, and
learned about new cultures
• The incorrect version gives all activities
equal emphasis, instead of making the last
three activities subordinate to the main
activity of traveling around the world
CAUSE – EFFECT PARALLELISM
Whenever there is a cause – effect in a
sentence, the effect must begin with an –ing
form of the verb after the comma
• The stock markets are strong, giving us
better forecasts
• The rains were heavy, leading to some
crop damage
• The principal declared the results, making
everyone cheer
PARALLELISM – 2
Verbs of Being – Be aware
Instead of expressing what the subject does,
these verbs express what a subject is, or the
condition a subject is in
Most common form of the verb being is “to
be”
Forms of “to be” are – is, am, are, was, were,
been, being
Other common words of being – appear,
become, feel, grow, look, remain, seem,
smell, sound, stay, taste, turn
• Incorrect: The flower bouquet WAS the
husband’s giving of love to his wife
• Correct: The flower bouquet WAS the
husband’s loving gift to his wife
• The two sides of the being word “was” are
flower bouquet and husband’s giving of
love. In order to achieve parallelism,
rewrite the sentence replacing giving with
the noun gift
Idioms with built-in parallel structure – the word immediately after the first part of the pair will
be the same as the word immediately following the second part of the pair (Example: Not only
in… but also in.. )
• More X than Y
• The more X the greater Y
• No less was X than was Y
• As X to Y
• Not only X but also Y
• Not X but rather Y
• X instead of Y
• The same to X as to Y
• Range from X to Y
• Both X and Y
• Mistake X for Y
• Prefer X to Y
• X regarded as Y
• To think of X as Y
• Believe X to be Y
• Between X and Y
• Either X or Y
• As X as Y
• Just as X so Y (Example: Just as gills are to fish, so lungs are to humans)
• Not so much X as Y (Example: Not so much to show Jane up as to make her appear foolish,
Sarah pointed out Jane’s error to their supervisor)
• More/greater/fewer less – than
• Same X as Y
• Just as X, so to Y (Example: Just as stealing is frowned upon, so too is cheating)
Articles are parallel – a, an, the
COMPARISON – 1
Category Example
Comparison signals
like, unlike, likening, more than, greater than, less than, shorter than, different from, as, as (adjective) as, as many as, as few as,
as much as, as little as, as high as, as short as
“Like” vs. “As”
Like should be used to compare people or things (any nouns)
 Incorrect: Bella and June, AS their mother, are extremely smart
 Correct: Bella and June, LIKE their mother, are extremely smart
As/as if should be used in a comparison involving clauses (any phrase that includes a verb)
 Incorrect: Just LIKE swimming is good exercise, skiing is a great way to burn calories
 Correct: Just AS swimming is good exercise, skiing is a great way to burn calories
Parallelism – must
compare similar things
and be structurally
parallel
 Incorrect: Frank’s build, like his brother, is broad (here, frank’s build is being compared to his brother. This is not logical as
it does not compare two similar things)
 Correct: Frank’s build, like that of his brother, is broad
 Incorrect: The host paid more attention to his business clients than others
 Correct: The host paid more attention to his business clients than to others
 Incorrect: Mary is taller than any girl in class (this implies that Mary is even taller than herself)
 Correct: Mary is taller than any other girl in class
 Incorrect: I enjoy flying planes more than I like to drive a car
 Correct: I enjoy flying planes more than I like driving cars
COMPARISON – 2
Category Example
Unclear comparisons
 Incorrect: Byron admired Dryden more than Wordsworth
 Correct: Byron admired Dryden more than he did Wordsworth
 Correct: Byron admired Dryden more than Wordsworth did
Incomplete comparisons
Incomplete comparisons are normally corrected by inserting a phrase like those of, those in, those at, that of, that in, and that at
 Incorrect: The peaches here are riper than other fruit stand (peaches are being compared with an unlike thing – fruit
stands)
 Correct: The peaches here are riper than those at other fruit stand
Like/Unlike
Compare similar things. The word after like/unlike must match the word after the comma
 Like Mary, John has a black car
 Unlike Mary, Susan is tall
Compared to vs.
compared with
Compared to (comparison between unlike things) vs. compared with (comparison between like things)
 He compared her to a summer day
 The police compared the forged signature with the original
Use comparative form when comparing two things; the superlative form when comparing 3 or more things
IDIOMS – 1
Correct Usage Incorrect Usage
Awareness of Awareness about
To regard X as Y To regard X as to regard Y
So X as to Y (emphasis is on X)
Example: So tall as to touch the ceiling
Enough X to Y (emphasis is on Y)
Example: Tall enough to touch the ceiling
Considered/consider
Example: “Scientists consider control factors an integral element…”
Considered to be
Example: “Scientists consider control factors to be an integral element…”
Declare XY
Example: Declared the Roman aqueduct of Segovia a Heritage of Humanity in
1985
Declare X to be Y
Example: Declared the Roman aqueduct of Segovia to be a Heritage of
Humanity in 1985
In the vicinity of Around the vicinity of
Prices can only be higher or lower This price is more expensive than that one
Agree with another person
Agree to/upon something
In contrast to In contrast with
Regarded as Regarded to be
When “rates” refer to the price charged, it should be followed by “for”
Example: “Rates for telephone service have increased in recent years.”
Thinking words such as “theory, belief ” or “believe” are often followed by
“that”
Example: Lucy’s belief that the Holocaust did not occur is misguided
Persuade X to do Y Persuade X from doing Y
IDIOMS – 2
Correct Usage Incorrect Usage
Apprenticeship as Apprenticeship of being
Dispute over Dispute about / dispute concerning
Attempted to Attempted
transparent enough to be
transparent as to be
transparent enough so as to be
Estimated to be Estimated at
Forbid X to do Y Forbid X from doing Y
Prefer X to Y Prefer X over Y
Consider XY
Many financial analysts consider an upward trend in a firm’s current ratio a sign of
improving liquidity
Consider X as Y
Many financial analysts consider an upward trend in a firm’s current ratio as a sign
of improving liquidity
Remembered for Remembered because of
ODDS AND ENDS – 1
RULE # 1 – Countable vs. uncountable
• Countable items include dollars, hats, people and buildings
• Uncountable items include money, water, wreckage and patience
- Always takes a singular verb in a sentence
Countable modifiers Uncountable modifiers Examples
Many Much
As many as As much as
Few/fewer Little/less
• Incorrect: The company fired no less than fifty employees
• Correct: The company fired no fewer than fifty employees
Number of Amount of
RULE # 2 – Among vs. Between
Relating to two things Relating to three or more things
Between X and Y Among X, Y and Z
X is better than Y X is the best (among x, y, and z)
X has more than Y X has the most (among x, y and z)
X has less than Y X has the least (among x, y and z)
ODDS AND ENDS – 2
Increase and decrease vs.
greater and less
Increase and decrease represent the change of ONE thing over time
Greater or less signals a comparison between TWO things
Greater than is correct only when it is used to describe numbers alone
(greater than 10). If a sentence measures a percent rather than solely a
number, the correct option is more than
 The price of silver increased by 10 dollars
 The price of silver is greater than the price of copper
Connecting words Be aware of sentences that have no logical connectors between 2
independent clauses. Common connecting words are: and, or, nor, but,
yet, although, when, because, for, since, before, after, if, & unless
 Incorrect: I need to relax, I have so much to do (run-on
sentence)
 Correct: I need to relax, but I have so much to do
Connecting punctuation The semicolon ( ; ) is used to connect two closely related statements.
BOTH statements need to stand together as independent sentences
The colon ( : ) is used to equate two parts of a sentence, where the
second part is dependent on the first part. It is often used to equate a
list with its components. One should be able to insert the word “namely”
after the colon. Only the statement that precedes the colon must be able
to stand alone
 Incorrect: Andrew and Lisa are inseparable; doing
everything together (second part of the sentence is
incapable of standing on its own)
 Correct: Andrew and Lisa are inseparable; they do
everything together
 Incorrect: I love listening to: classical rock, rock and pop
music
 Correct: I love many kinds of music: [namely] classical rock,
rock and pop music
If/Whether Whether is correct when you're discussing two options (whether to get
chocolate or strawberry ice cream). On the GMAT, whether will (almost)
always beat if
Only/once Words with only/once always refer to the item coming immediately after
them
 Incorrect: Existed once
 Correct: Once existed
ODDS AND ENDS – 3
Because vs. In that When ‘because’ and ‘in that’ are in a sentence, more often than not,
‘in that’ would be correct. ‘In that’ qualifies the previous sentence,
while ‘because’ is just used to show a simple causal relationship
Teratomas are unusual forms of cancer because they are composed of
tissues such as tooth and bone not normally found in the organ in
which the tumor appears.
A. because they are composed of tissues such as tooth and bone
B. because they are composed of tissues like tooth and bone that are
C. because they are composed of tissues, like tooth and bone, tissues
D. in that their composition , tissues such as tooth and bone, is
E. in that they are composed of tissues such as tooth and bone,
tissue
Can vs. Could If you are just assuming something, ‘could’ should be used
Could is used for:
- Possibility (John could be the one who stole the money)
- Condition (If I had more time, I could travel around the world)
- Suggestion (You could spend your vacation here)
- Polite request (Could I have something to drink?)
Artificial intelligence emerged during the late 1950's as an academic
discipline based on the assumption that computers are able to be
programmed to think like people
A. are able to be programmed to think like people
B. were able to be programmed to think as people
C. can be programmed to think as people can
D. could be programmed to think like people
E. are capable of being programmed to think like people do
Usual vs. Is usual When something is compared to a subgroup to which it belongs, is
usual should be used. When something is compared to itself, usual is
fine
 Incorrect: A Mercedes is more expensive than usual for a car
 Correct: A Mercedes is more expensive than is usual for a car
 He is nicer than usual
The If we use ‘the’ we are saying that there are only 50 million Turks in
the whole world; if we don’t use ‘the’ we’re saying that there are
possibly more than 50 million Turks in the world
• Although about 99% of the more than 50 million Turks are Muslims
• Although about 99% of more than 50 million
MISCELLANEOUS
• “Because of” is used to modify an adverbial phrase (verb/verb phrase)
• “Despite” requires a noun or a noun phrase
• Just as can replace “in the same way that”
• "instead of" is reserved for nouns and "rather than" for verbs
THINGS TO AVOID
• Noun forms (words ending in -ance, -ence, -ment, -ion,-ity)
• Because of
• Apostrophe
• Passive voice
WORDS/PHRASES THAT ARE ALMOST ALWAYS INCORRECT
• Do it (use do so)
- Incorrect: He said that I ate his cookies, but I didn’t do it
• The numbers of
- Incorrect: The politicians were amazed at the numbers of anti-war protestors
• Whether or not (just use whether)
- Incorrect: He couldn’t decide whether or not to apply to Harvard
• Due to (should only be used if it can be replaced by ‘caused by’; used to modify a noun)
- Incorrect: the game was postponed due to rain
- Correct: His failure was due to his laziness
- Incorrect: He failed due to his laziness
• So as to (use such that)
• Just like
• On account of
• Occurring (to be used only for natural calamity)
• Such like
• Owing to
• Lesser
• Hopefully
• Seeming
• Not any
• Being/ having
CRITICAL REASONING
IMPORTANT NOTES
• When reading Logical Reasoning stimuli, you should seek to make several key determinations:
– Does the stimulus contain an argument or is it only a set of factual statements?
– If the stimulus contains an argument:
• What is the conclusion?
• Is the argument strong or weak? To determine the strength of the argument, consider the relationship between the premises
and the conclusion—do the premises strongly suggest that the conclusion would be true? Does the conclusion feel like an
inevitable result of the premises? Or does the conclusion seem to go beyond the scope of the information in the premises?
DO THE FACTS SUPPORT THE CONCLUSION?
– Read the fine print!
• Quantity and Probability Indicators
• Scope
– Carefully read and identify the question stem. Do not assume that certain words are automatically associated with certain question
types
– One of the most effective techniques for quickly finding correct answer choices and avoiding incorrect answer choices is pre-
phrasing. Pre-phrasing an answer involves quickly speculating on what you expect the correct answer will be based on the
information in the stimulus
• INFER = MUST BE TRUE
PREMISE ARGUMENT
STIMULUS
Indicators
Definition
A fact, proposition, or statement from which a
conclusion is made
Premises support and explain the conclusion
Example: The Jacksonville area has just over one
million residents. The Cincinnati area has almost
two million residents. The New York area has
almost twenty million residents
because, since, for, for example, for the reason that,
in that, given by, as indicated by, due to, owing to,
this can be seen from, we know this by
Additional premise indicators (another premise
that supports the conclusion but is sometimes non-
essential to the conclusion) – furthermore,
moreover, besides, in addition, what’s more
thus, therefore, hence, consequently, as a result, so,
accordingly, clearly, must be that, shows that,
conclude that, follows that, for this reason
Premise + Conclusion
Conclusion is a statement or judgment that follows
from one or more reasons
Conclusion = premise + assumption (implicit)
Example: All professors are ethical. Mason is a
professor. So Mason is ethical.
The first two statements in this argument give the
reasons (or “premises”) for accepting the third
statement, which is the conclusion
Imp: Premises and conclusions can be presented in any order; the order has no effect on the logical
structure of the argument. Similarly, it is not necessary for the premise and conclusion to be split into
multiple sentences
Tip: The conclusion is generally the first or last sentence of a stimulus
DETERMINING PREMISES AND CONCLUSIONS
Example Premise Conclusion (may be added in the question
stem )
“The rapid diminishment of the ecosystem of the
Amazon threatens the entire planet. Consequently, we
must take immediate steps to convince the Brazilian
government that planned development projects need
to be curtailed for the simple reason that these
development projects will greatly accelerate the loss of
currently protected land.”
Premise 1: “The rapid diminishment of the ecosystem of
the Amazon threatens the entire planet.”
Premise 2: “for the simple reason that these development
projects will greatly accelerate the loss of currently
protected land.”
We must take immediate steps to convince the
Brazilian government that planned development
projects need to be curtailed
Every professor at Fillmore University teaches exactly
one class per semester. Fillmore’s Professor Jackson,
therefore, is teaching exactly one class this semester.
Moreover, I heard Professor Jackson say she was
teaching only a single class
Every professor at Fillmore University teaches exactly one
class per semester
The first sentence is the main proof offered by the author
for the conclusion. The third sentence begins with the
additional premise indicator “moreover.” The premise in
this sentence is non-essential to the argument, but
provides additional proof for the conclusion and could be,
if needed, used to help prove the conclusion separately
(this would occur if an objection was raised to the first
premise)
Fillmore’s Professor Jackson, therefore, is teaching
exactly one class this semester
Humans cannot live on Venus because the surface
temperature is too high
The surface temperature is too high Humans cannot live on Venus
Therefore, since higher debt has forced consumers to
lower their savings, banks now have less money to loan
Higher debt has forced consumers to lower their savings Banks now have less money to loan
Television has a harmful effect on society. This can be
seen from the poor school performance of children
who watch significant amounts of television and from
the fact that children who watch more than six hours of
television a day tend to read less than non-television
watching children
Premise 1: This can be seen from the poor school
performance of children who watch significant amounts of
television
Premise 2: and from fact that children who watch more
than six hours of television a day tend to read less than
non-television watching children
Television has a harmful effect on society.
TYPES OF CRITICAL REASONING QUESTIONS
PROVE HELP HURT
Must accept the stimulus – even if it contains an
error in reasoning – and use it to prove that one
of the answer choices must be true
Any information in an answer choice that does
not appear either directly in the stimulus or as a
combination of the items in the stimulus will be
incorrect
Reasoning errors are often present in the
stimulus; you may use the “new” information
provided in the answer choices to
strengthen/support the argument
Reasoning errors are often present in the
stimulus; you may use the “new” information
provided in the answer choices to weaken the
argument
Must be true/ Inference Assumption Weaken
Main point Strengthen/support
Method of reasoning Paradox
Flaw in reasoning
Parallel reasoning
A. MUST BE TRUE/ INFERENCE
Paraphrased answers - answers that restate a portion of
the stimulus in different terms. Because the language is
not exactly the same as in the stimulus, paraphrased
answers can be easy to miss
CORRECT ANSWERS
Combination answers - Answers that result from
combining two or more statements in the stimulus
Things to keep in mind
- The stimulus comprises a set of facts ONLY, and the
objective is to determine the conclusion based on the
facts presented. NO ADDITIONAL ASSUMPTIONS MUST BE
MADE
- Be careful of indicator and modifier words such as
entirely, some, almost, all, could lead to, many , not, but .
They define the scope (narrow or broad) and the tone of
the argument.
INCORRECT ANSWERS
Answers that are
possible but not
certain
Because the criteria in the question stem requires you to find an answer choice that Must Be True, answers that only could be true or are even
likely to be true are incorrect. These answers are attractive because there is nothing demonstrably wrong with them (for example, they do not
contain statements that are counter to the stimulus)
Exaggerated
answers
These answers take information from the stimulus and then stretch that information to make a broader statement that is not supported by the
stimulus
For example, if the stimulus stated that “Some software vendors recently implemented more rigorous licensing procedures”, an incorrect
answer would exaggerate one or more of the elements: “Most software vendors recently implemented more rigorous licensing procedures.”
Answers that
present so-called
“new information”
For example, if the stimulus discusses the economic policies of Japan, be careful with an answer that mentions U.S. economic policy. Look
closely at the stimulus—does the information about Japanese economic policy apply to the U.S.?
Do not eliminate if the answer falls within the sphere of a term or concept in the stimulus, or if it is a consequence of combining stimulus
elements
Reverse answers The stimulus might state, “Many people have some type of security system in their home.”
An incorrect answer then reverses the elements: “Some people have many types of security systems in their home.”
Shell Game Occurs when an idea or concept is raised in the stimulus and then a very similar idea appears in the answer choice, but the idea is changed just
enough to be incorrect but still attractive
Opposite answer Opposite of the stated facts of the stimulus. Example, stimulus shows that X caused Y, but answer choice states that Y caused X
When a stimulus contains only the opinions of others, then in a Must Be True question you can eliminate any answer choice that makes a flat assertion without
reference to those opinions
MUST BE TRUE – CONTINUED
Flavonoids are a common component of almost all plants, but a specific variety of
flavonoid in apples has been found to be an antioxidant. Antioxidants are known to be a
factor in the prevention of heart disease
Which one of the following can be properly inferred from the passage?
A. A diet composed largely of fruits and vegetables will help to prevent heart disease
B. Flavonoids are essential to preventing heart disease
C. Eating at least one apple each day will prevent heart disease
D. At least one type of flavonoid helps to prevent heart disease
E. A diet deficient in antioxidants is a common cause of heart disease
1st Statement: Flavonoids are a common component of almost all plants
2nd Statement: A specific variety of flavonoid in apples has been found to be an antioxidant
3rd Statement: Antioxidants are known to be a factor in the prevention of heart disease
A The answer choice could be true, but it is too broad to be supported by the facts: nowhere
are we told that a diet of fruits and vegetables will help prevent heart disease. Perhaps
apples are the only fruit with the antioxidant flavonoid and there is nothing beneficial
about other fruits and vegetables. And, eating a diet of fruits and vegetables is no
guarantee that the diet includes apples. Regardless, this answer choice can be especially
attractive because it plays on the general perception that fruits and vegetables are good
for you
B Nothing in the stimulus supports the rather strong statement that flavonoids are essential
to preventing heart disease
C This answer is incorrect because the language is too strong: the stimulus only stated that
apples contain an element that was a factor in preventing heart disease, not that they
definitely will prevent heart disease
D This answer is the closest to our pre-phrase, and this is the correct answer. Notice how the
language of this answer choice—“helps to prevent”—matches the stimulus language—
“factor in the prevention.”
E This answer choice also could be true, but it cannot be correct because the stimulus makes
no mention of the causes of heart disease. Just because an antioxidant can help prevent
heart disease does not mean that a lack of antioxidants causes heart disease
B. MAIN POINT
Example:
Journalist: A free marketplace of ideas ensures that all ideas get a fair hearing. Even ideas tainted with prejudice and malice can prompt beneficial outcomes. In most countries, however, the
government is responsible for over half the information released to the public through all media. For this reason, the power of governments over information needs to be curtailed. Everyone
grants that governments should not suppress free expression, yet governments continue to construct near monopolies on the publication and dissemination of enormous amounts of
information
Which one of the following most accurately expresses the conclusion of the journalist’s argument?
(A) The freedom of the marketplace of ideas is in jeopardy.
(B) Preserving a free marketplace of ideas is important
(C) The control that governments have over information needs to be reduced
(D) Ideas that have malicious content or stem from questionable sources can be valuable
(E) Governments have near monopolies on the dissemination of many kinds of information
(A) The author would agree with this statement but this is not the Main Point of the argument; rather, it is closer to a premise that might support the conclusion
(B) The author believes that the freedom of the marketplace of ideas is at risk, and in stating that we should curtail the government’s power over information, the author assumes that
preserving a free marketplace of ideas is important. Thus this answer choice would be better described as an unstated premise that supports the conclusion
(C) This is the correct answer. Remember, any answer that is a paraphrase of the conclusion of the argument will be the correct answer to a Main Point question
(D) The stimulus specifically notes that malicious or prejudicial ideas can “prompt beneficial outcomes.” The outcome of an idea is different than stating the ideas themselves “can be
valuable.”
(E) The stimulus states that “governments continue to construct near monopolies on the publication and dissemination of enormous amounts of information.” This phrasing is not the same as
answer choice (E), which asserts that the government already has a monopoly on the dissemination of many kinds of information
Primary objective
Question identifiers
Incorrect answers
Answers that are true but do not encapsulate the author’s point
Answers that repeat premises of the argument
“Which one of the following most accurately expresses the main conclusion of the argument?”
“Which one of the following most accurately restates the main point of the argument”
To identify the CONCLUSION based ONLY on facts/premises explicitly listed in the stimulus
C. WEAKEN
Question Identifiers “Which of the following most seriously undermines the argument?”
“Which of the following, if true, calls into question the validity of the argument?”
Trigger words: Weaken, attack, undermines, contradicts, evidence against, challenge, damage, counter, refute etc.
Primary Objective There is always a flaw in the reasoning of the stimulus (gross generalization, conclusions drawn from incomplete information, improper
comparison etc.)
To solve these questions, you first need to identify the premise and the conclusion. In this question type, we assume an answer choice
presented to be true – even if it introduces new information (obviously, the information has to be relevant to the stimulus)
Incorrect answers • Opposite answers – answers that strengthen rather than weaken the argument
• Shell game answers – similar idea to that of the stimulus, but not entirely true
• Improper comparison – comparing two or more items that are essentially different
• Out of scope answers
• Wrong tone answers
• Reversal of causality/incorrect causality
Cause & Effect
Reasoning
- Causality occurs when one event is said to make another occur. Just because one event occurs before the other, or that two events occur
simultaneously, does not mean that there is a cause-effect relationship
- Words that indicate cause-effect relationship: cause by, because of, responsible for, reason for, leads to, induced by, promoted by,
determined by, produced by, product of, played a role in, was a factor in, is an effect of
- The question assumes that the only cause is the one stated in the conclusion and that there are no other causes that can create that
particular effect
- In weaken questions, the correct answer choice with do any of the following:
• Find an alternate cause for the stated effect. Because the author believes there is only one cause, identifying another cause
weakens the conclusion
• Show that even when the cause occurs, the effect does not occur
• Show that although the effect occurs, the cause did not occur
• Show that the stated relationship is reversed (the claimed effect is actually the cause of the claimed cause)
• Show that a statistical problem exists with the data used to make the casual statement
Example
The consumption of ice cream has been found to correlate with the murder rate. Therefore, consuming ice cream must cause one to be more
likely to commit murder
However, this argument fails to take into account that a third element, hot weather, could be the cause of the increase in murder rate and in
the consumption of ice cream
WEAKEN – CONTINUED
Robot satellites relay important communications and identify weather
patterns. Because the satellites can be repaired only in orbit, astronauts are
needed to repair them. Without repairs, the satellites would eventually
malfunction. Therefore, space flights carrying astronauts must continue.
Which of the following, if true, would most seriously weaken the argument
above?
A. Satellites falling from orbit because of malfunctions burn up in the
atmosphere
B. Although satellites are indispensable in the identification of weather
patterns, weather forecasters also make some use of computer
projections to identify weather patterns
C. The government, responding to public pressure, has decided to cut the
budget for space flights and put more money into social welfare
programs
D. Repair of satellites requires heavy equipment, which adds to the
amount of fuel needed to lift a spaceship carrying astronauts into orbit
E. Technical obsolescence of robot satellites makes repairing them more
costly and less practical than sending new, improved satellites into orbit
A Irrelevant
B Doesn’t provide a reason for not sending astronauts to space. Out of scope
C Though this seems like a right answer, think about it. This is lots of irrelevant
information, and doesn’t answer our question directly
D Fuel? This is a shell game fallacy. Clearly shows a statement that could be
true and is very attractive to the test taker, but irrelevant to what’s asked
E This makes sense. If repair cost > new satellite cost, why send astronauts to
space to repair them? Just send new satellites
Violent crime in this town is a becoming a serious problem. Compared to
last year, local law enforcement agencies have responded to 17 per cent
more calls involving violent crimes, showing that the average citizen of this
town is more likely than ever to become a victim of a violent crime.
Which one of the following, if true, most seriously weakens the argument?
A. The town’s overall crime rate appears to have risen slightly this year
compared to the same period last year
B. In general, persons under the age of 65 are less likely to be victims of
violent crimes than persons over the age of 65
C. As a result of the town’s community outreach programs, more people
than ever are willing to report violent crimes to the proper authorities
D. In response to worries about violent crime, the town has recently
opened a community centre providing supervised activities for
teenagers
E. Community officials have shown that a relatively small number of
repeat offenders commit the majority of violent crimes in the town
A This is an opposite answer that strengthens the argument
B Because the argument is about “the average citizen of this town,”
information about victims of a certain age is irrelevant
C This is the correct answer. By showing that people are more willing to
report crimes (and thus call them in for response), an alternate cause for
the rise in the number of calls is given
D This answer only addresses an effect of the concern over crime, and does
not address the causal relationship that underlies the argument
E This answer does not address a possible rise in crime or the reasons for the
rise in responses to calls involving violent crime
D. STRENGTHEN
Question Identifiers Stem would indicate some kind of “support” relationship – strengthen, justify, help, support etc.
** “The stimulus supports which of the following” would be an inference/main point question as opposed to the “strengthen” question
which would point to “the answer choice supporting the stimulus” – subtle but important difference
Primary Objective Identify the conclusion—this is what you are trying to strengthen!
Find the missing link between a premise and the conclusion (typically an assumption)
Important to keep in
mind
Don’t try to disprove a conclusion
Incorrect answers • Opposite answers – answers that weaken rather than strengthen the argument
• Out of Scope
• Shell Game
• Wrong tone answers
• Answers that merely state the premise
Correct answers
• Connect evidence with conclusion better
• Make the conclusion stronger
• Arguments that contain analogies or use surveys rely upon the validity of those analogies and surveys. Answer choices that strengthen
the analogy or survey, or establish their soundness, are usually correct
• Strengthen the evidence with new information (perhaps an assumption is needed to make the argument work
STRENGTHEN – CONTINUED
Dr. Larson: Sleep deprivation is the cause of many social ills, ranging from
irritability to potentially dangerous instances of impaired decision making.
Most people suffer from sleep deprivation to some degree. Therefore we
should restructure the work day to allow people flexibility in scheduling
their work hours
Which of the following, if true, would most strengthen the medical doctor’s
argument about sleep deprivation?
The conclusion of the argument is straightforward. Allow flexibility in
scheduling work hours. The basic underlying assumption in the conclusion
basically tells you that the author is assuming that greater flexibility would
promote better sleeping times. Now we can have this in our mind as we
look for answers.
A. The primary cause of sleep deprivation is overwork
B. Employees would get more sleep if they had greater latitude in
scheduling their work hours
C. Individuals vary widely in the amount of sleep they require
D. More people would suffer from sleep deprivation today than did in the
past if the average number of hours worked per week had not
decreased
E. The extent of one’s sleep deprivation is proportional to the length of
the workday
A Perfect shell game choice. So this choice is telling you that overwork causes
sleep deprivation. We are not talking about the quantity of work here, but
instead the flexibility of scheduling. Irrelevant
B Correct
C Completely irrelevant. We are not talking about how much sleep an
individual requires at all. It presents new information but without any
connection
D This talks about a decrease in work hours and in a very convoluted
structure. Are we talking about the number of hours worked? No. We are
talking only about the flexibility of scheduling
E Once again, length is immaterial
E. ASSUMPTIONS
Question Identifiers “The conclusion depends on which of the following?”
“The author assumes that?”
“Based on”
“Assumption Made”
“Cannot be true unless?”
Primary Objective An assumption is an unstated premise and is NECESSARY to arrive at a conclusion
Find the missing link between a premise and the conclusion (typically an assumption)
Incorrect answers • Contain extra information - For example, let’s say that an argument requires the assumption that “all dogs are intelligent”. The
correct answer could be that statement or even a subset statement such as “all black dogs are intelligent” or “all large dogs are
intelligent”. However, in the case of the statement “all dogs and cats are intelligent”, the additional information about cats is not part
of the authors assumption, and would make the answer choice incorrect
• Answers that claim an idea was the most important consideration for the author
• Answer choice that repeats the premise
Difference between
Strengthen & Assumption
Questions
• Strengthen questions ask you to support the argument in any way possible. This type of answer has great range, as the additional
support provided by the answer choice could be relatively minor or major. Speaking in numerical terms, any answer choice that
strengthens the argument, whether by 1% or by 100%, is correct
• Assumption questions ask you to identify a statement that the argument assumes or supposes. An assumption is simply an unstated
premise— what must be true in order for the argument to be true. An assumption can therefore be defined as what is necessary for
the argument to be true
• Example: An argument concludes that a teenager is an outstanding golfer. In an Assumption question, the correct answer could be:
“The teenager always hits the ball” or “The teenager never swings and misses the ball.” Either statement is an assumption of the
argument; otherwise how could the teenager be an outstanding golfer? In a Strengthen question, the correct answer could be: “The
teenager won a local club tournament.” This answer choice supports the idea that the teenager is an outstanding golfer, but does not
undeniably prove the teenager to be outstanding (what if the tournament was composed primarily of pre-teen players?) nor is the
answer an assumption of the conclusion
CORRECT ASSUMPTION ANSWER CHOICE
SUPPORTER
• Links together new/ rogue elements in the stimulus
• Conclusion often contains a piece of information not previously included
in the stimulus
• Fills logical gaps in the argument
Example: All male citizens of Athens had the right to vote. Therefore,
Socrates had the right to vote in Athens
Assumption: Socrates was male
DEFENDER
Eliminate ideas or assertions that would undermine the conclusion. In this
sense, they “defend” the argument by showing that a possible source of
attack has been eliminated
Example: People who read a lot are more intelligent than other people.
Thus, reading must cause a person to be intelligent
Assumption: All other alternative explanations (such as regular exercise,
high-protein diet, genetics etc.) are assumed not to exist
Watch for answers starting with the phrase “at least one” or “at least some.” For some reason, when an Assumption answer choice starts with either of the
above constructions the chances are unusually high that the answer will be correct
ASSUMPTION NEGATION TECHNIQUE
Basically involves converting the assumption question into a weaken question. The technique can only be applied to assumption questions, so you need to be
careful, but once you’ve eliminated other possible answers, follow this technique to check the remaining options:
• Negate the answer choice – This basically asks you to assume the opposite of whatever is given in the answer choice ( I went to the beach yesterday will
become I did not go to the beach everyday last week; sweet will become not sweet; all the missions succeeded will become not all of the missions
succeeded)
** All = 100; Not All = 0-99; Some = 1-100; None = 0
• Does the negated answer choice make the conclusion collapse? – If the answer is yes, then the answer choice is the right one. Any negated answer choice
that attacks the conclusion or questions its validity is one that is the right answer.
F. RESOLVE THE PARADOX
Question Identifiers “Which of the following, if true, most helps resolve the apparent paradox?”
Resolve/ reconcile/ Explain the following – Discrepancy/ contradiction/ conflict/ puzzle
Primary Objective Two contradictory facts are presented as they are. There is no inference/conclusion. You must resolve the paradox (i.e., allow both sides
to be factually correct)
Incorrect answers • Answer explains only one side of the paradox
• Does not conform to the specifics of the stimulus
• If the stimulus contains a paradox where two items are similar, then an answer choice that explains a difference between the two
cannot be correct and vice versa. The answer choice must resolve the paradox, not strengthen or weaken it
Correct Answers Correct answers will actively resolve the paradox, that is, it will allow both sides to be factually correct and it will either explain how the
situation came into being or add a piece of information that shows how the two ideas or occurrences can co-exist
Park Ranger: When snowfall levels are below average during winter months, scattered patches on the forest floor often remain exposed and accessible to
scavenging wildlife. Because squirrels are able to collect nuts only in the snow-free areas of the forest, the squirrel population tends to increase when there is a
below average snowfall. However after last year’s unprecedented snow-free winter season, the squirrel population in this region was determined to be a 20
year low
Which of the following, if true, helps explain the paradox above?
A. When snowfall is above average, squirrel populations tend to diminish as squirrels are unable to forage for food in snow-covered areas
B. The squirrels’ spring breeding season does not begin until all of the snow in the forest has melted.
C. The red-tailed hawk, the squirrel’s most common predator, does not migrate south out of the forest until the first snowfall of the winter season.
D. Forest squirrels rarely feed on berries or fruits and prefer nuts for their high calorific content
E. The current system of estimating squirrel population size is thought to be extremely accurate in its projections
A Irrelevant. The first phrase itself tells us this is useless since we are looking to explain an event that happens during a snow-free winter
B This doesn’t really explain anything either. If at all anything, the population should have gone up.
C This seems good. If the predator doesn’t move until the snowfall, and there was no snowfall, then the squirrel population would have gone down
D Irrelevant
E Irrelevant
G. BOLD FACED QUESTIONS
Question Identifiers “The two bold faced sentences play which of the following roles?”
Primary Objective These questions are heavily dependent on interpreting the premise and identifying the conclusion of the stimulus correctly and hence can
be considered similar to an inference/must be true question type. To correctly answer the question it’s important to identify the pattern of
reasoning in the stimulus correctly. Thus, going back to the inference lesson, for this question type, we need to isolate the conclusion,
understand the premise and identify any logical errors in the drawing of the conclusion
Incorrect answers • Answer explains only one side of the bold faced questions
• Does not conform to the specifics of the stimulus
Historian: Newton developed mathematical concepts and techniques
that are fundamental to modern calculus. Leibniz developed closely
analogous concepts and techniques. It has traditionally been thought
that these discoveries were independent. Researchers have however,
recently discovered notes of Leibniz’ that discuss one of Newton’s
books on mathematics. Several scholars have argued that since the
book includes a presentation of Newton’s calculus concepts and
techniques and since the notes were written before Leibniz’ own
development of calculus concepts and techniques, it is virtually certain
that the traditional view is false. A more cautious conclusion than this is
called for, however. Leibniz’ notes are limited to early sections of
Newton’s book; sections that precede the ones in which Newton’s
calculus concepts and techniques are presented.
In the historian’s reasoning, the two boldfaced portions play which of
the following roles?
This is a question where it’s absolutely necessary to identify what the
author’s stand on the argument is, and hence identifying the
conclusion becomes vital. The traditional view is that Leibniz and
Newton developed these simultaneously. But people believe this is
false because they found Leibniz mentioning that he’d studied
Newton’s book. Look at what the historian says in response: A more
cautious conclusion. So this means he doesn’t believe that Leibniz had
written the works after Newton and hence justifies that by giving an
alternative explanation.
So to deconstruct this question we have two statements:
Researcher’s conclusion: Traditional view is false.
Historian’s conclusion: Researcher’s conclusion is false.
A The first provides evidence in support of the overall position that the
historian defends; the second is evidence that has been used to support an
opposing position.
The historian doesn’t defend the position, he opposes it. So incorrect
B The first provides evidence in support of the overall position that the
historian defends; the second is that position
Once again, the historian doesn’t defend this position, hence incorrect.
C The first provides evidence in support of an intermediate conclusion that is
drawn to provide support for the overall position that the historian defends;
the second provides evidence against that intermediate conclusion
This is the kind of complicated wording that GMAT likes to trick us with.
There is no intermediate conclusion, and the historian doesn’t support the
overall conclusion
D The first is evidence that has been used to support a conclusion that the
historian criticizes; the second is evidence offered in support of the
historian’s own position
This is true. The historian opposes the first and gives an alternative
explanation.
E The first is evidence that has been used to support a conclusion that the
historian criticizes; the second is further information that substantiates that
evidence
The second part of this is wrong. The first might be evidence that supports
something the historian criticizes, however the second part is his alternative
explanation and is not in support of the first. Hence incorrect
H. METHOD OF REASONING
Question Identifiers • “The argument proceeds by”
• “The argument derives its conclusion by”
• “Which one of the following is an argumentative strategy employed in the argument?”
• “The argument employs which one of the following reasoning techniques?”
Primary Objective Focus on the form of the argument instead of the concrete facts
Incorrect answers • Describe an event that did not occur in the stimulus/ “New” Element Answers
• Half right/half wrong – start by describing something that occurred in the argument and end by describing something that didn’t
• Exaggerated/opposite/reversed answers
Garbage in this neighbourhood probably will not be collected until
Thursday this week. Garbage is usually collected here on Wednesdays, and
the garbage collectors in this city are extremely reliable. However, Monday
was a public holiday, and after a public holiday that falls on a Monday,
garbage throughout the city is supposed to be collected one day later than
usual. The argument proceeds by:
(A) treating several pieces of irrelevant evidence as though they provide
support for the conclusion
(B) indirectly establishing that one thing is likely to occur by directly ruling
out all of the alternative possibilities
(C) providing information that allows application of a general rule to a
specific case
(D) generalizing about all actions of a certain kind on the basis of a
description of one such action
(E) treating something that is probable as though it were inevitable
As usual, we begin by analyzing the structure of the problem:
Premise: Garbage is usually collected here on Wednesdays, and the
garbage collectors in this city are extremely reliable
Premise: Monday was a public holiday
Premise: After a public holiday that falls on a Monday, garbage throughout
the city is supposed to be collected one day later than usual
Conclusion: Garbage in this neighbourhood probably will not be collected
until Thursday this week
The argument is sound and the conclusion seems reasonable. The language
in the conclusion is not absolute (“probably”), and this is justified since the
language used in the argument—“usually” and “supposed to be”—is also
probabilistic. Knowing that the argument is valid, the question you must
ask yourself is, “How would I describe the structure of this argument?”
A. This answer forces you to make an assessment of the premises (the “evidence”) as
they relate to the conclusion. Are the premises irrelevant to the conclusion? Clearly
not. Therefore, this answer is incorrect
B. This is a half-right, half-wrong answer. The argument does establish “that one thing
is likely to occur.” But, is this established by ruling out all of the alternative
possibilities? No, to do that would mean presenting arguments against the garbage
being collected on Friday, Saturday, Sunday, etc. Since this section of the answer
choice does not occur, this answer is incorrect. Also, because the argument does
not rule out all the alternatives, the conclusion is not established “indirectly.”
C. This is the correct answer. Consider each piece of the argument: “providing
information”—a variety of information about the garbage situation is provided.
“application of a general rule”—the general rule is that “After a public holiday that
falls on a Monday, garbage throughout the city is supposed to be collected one day
later than usual.” “to a specific case”—the specific case is the pickup of garbage this
week in this neighbourhood. Given that all elements occurred and the answer
presents an accurate description of the way the author made his or her argument,
this answer is correct. Now, take a moment and compare this answer to the pre-
phrase you made after reading the stimulus. How similar are the two? Given that
you may not be familiar with the language used by the test makers, the two may
not be very similar. For example, note the use in this answer of “general rule” to
describe the last sentence of the stimulus. The test makers could also have used a
phrase like “basic principle” to achieve the same result. Your job is to match their
language to what occurred in the stimulus
D. This answer is an overgeneralization—a situation where one instance is used to
make a broad based conclusion. This is a Reverse Answer since the stimulus actually
uses a general principle and applies it to one instance
E. This is an Exaggerated Answer. The conclusion states that “Garbage in this
neighbourhood probably will not be collected until Thursday this week” and the use
of “probably” is a clear and obvious indication that the author does not think the
Thursday garbage pickup is inevitable
I. FLAW IN REASONING
Question
Identifiers
“The reasoning in the argument is most vulnerable to criticism on the ground that the argument
“The reasoning above is flawed because it fails to recognize that”
“The reasoning in the argument is fallacious because the argument”
Primary Objective Flaw in the Reasoning questions are exactly the same as Method of Reasoning questions with the important exception
that the question stem indicates that the reasoning in the stimulus is flawed. Because the question stem reveals that a
flaw is present, you need not make a determination of the validity of the stimulus
COMMON ERRORS OF REASONING
Causal conclusion vs.
causal claim made in
the premise
If the causal statement is the conclusion, then the reasoning is
flawed
Premise: In North America, people drink a lot of milk
Premise: There is a high frequency of cancer in North America
Conclusion: Therefore, drinking milk causes cancer
In this case, the author takes two events that occur together and
concludes that one causes the other
If the causal statement is the premise, then the argument may be
flawed, but not because of the causal statement
Premise: Drinking milk causes cancer
Premise: The residents of North America drink a lot of milk
Conclusion: Therefore, in North America there is a high frequency
of cancer among the residents
**Uncertain Use of a
Term or Concept
“Some people claim that the values that this country was built on
are now being ignored by modern-day corporations. But this is
incorrect. Corporations are purely profit driven enterprises,
beholden only to their shareholders, and as such they can only
assess objects based on their value.”
The term “value” is used in the example above in two different
senses: first in a moral or ethical sense and then in a monetary
sense. This shift in meaning undermines the author’s position
- “depending on the ambiguous use of a key term”
- “it confuses two different meanings of the word ‘solve’
- “relies on interpreting a key term in two different ways”
- “equivocates with respect to a central concept”
- “allows a key term to shift in meaning from one use to the next”
- “fails to define the term”
COMMON ERRORS OF REASONING – 2
Source Argument Attacks the person (or source) instead of the argument they
advance. Because the GMAT is concerned solely with argument
forms, a speaker can never validly attack the character or motives
of a person; instead, a speaker must always attack the argument
advanced by a person.
Here is an example: “The anti-smoking views expressed by
Senator Smith should be ignored. After all, Smith himself is a
smoker!”
A source argument can take different forms, including the
following:
1. Focusing on the motives of the source
2. Focusing on the actions of the source (as in the above
example)
“makes an attack on the character of opponents”
“it is directed against the proponent of a claim rather than against the
claim itself”
“he directs his criticism against the person making the argument rather
than directing it against the argument itself”
“it draws conclusions about the merit of a position and about the
content of that position from evidence about the position’s source”
“assuming that a claim is false on the grounds that the person
defending it is of questionable character”
Circular Reasoning In circular reasoning the author assumes as true what is supposed
to be proved
Example: “This essay is the best because it is better than all the
others.”
In this example the premise and the conclusion are identical in
meaning. As we know, the conclusion should always follow from
the premise. In the example above, the premise supports the
conclusion, but the conclusion equally supports the premise,
creating a “circular” situation where you can move from premise
to conclusion, and then back again to the premise
“it assumes what it seeks to establish”
“argues circularly by assuming the conclusion is true in stating the
premises”
“presupposes the truth of what it sets out to prove”
“the argument assumes what it is attempting to demonstrate”
“it takes for granted the very claim that it sets out to establish”
“it offers, in place of support for its conclusion, a mere restatement of
that conclusion”
Errors of Conditional
Reasoning
The authors can either mistake a necessary condition for a
sufficient condition, or mistake a sufficient condition for a
necessary condition
“it treats something that is necessary for bringing about a state of
affairs as something that is sufficient to bring about a state of affairs”
“confuses a sufficient condition with a required condition”
COMMON ERRORS OF REASONING – 3
Cause and effect - Assuming a causal relationship on the basis of the sequence
of events
- Assuming a causal relationship when only a correlation
exists
- Failure to consider an alternate cause for the effect, or an
alternate cause for both the cause and the effect
- Failure to consider that the events may be reversed
- “mistakes a temporal relationship for a causal relationship”
- “confusing the coincidence of two events with a causal relation
between the two”
- “assumes a causal relationship where only a correlation has been
indicated”
- “fails to exclude an alternative explanation for the observed effect”
- “the author mistakes an effect for a cause”
Straw man This error occurs when an author attempts to attack an
opponent’s position by ignoring the actual statements made by
the opposing speaker and instead distorts and refashions the
argument, making it weaker in the process. Often this error is
accompanied by the phrase “what you’re saying is” or “if I
understand you correctly,” which are used to preface the
refashioned and weakened argument
- “refutes a distorted version of an opposing position”
- “ incorrectly describing the student representative’s position,
thereby making it easier to challenge”
- “portrays opponents’ views as more extreme than they really are”
- “distorts the proposal advocated by opponents”
General Lack of
Relevant Evidence
for the Conclusion
Failure to provide any information to support the conclusion/
irrelevant information
Example: “Some critics claim that scientific progress has increased
the polarization of society and alienated large segments of the
population. But these critics are wrong because even a cursory
glance at the past shows that society is always somewhat
polarized and some groups are inevitably alienated.”
- “The author cites irrelevant data.”
- “draws a conclusion that is broader in scope than is warranted by
the evidence advanced”
- “It uses irrelevant facts to justify a claim about the quality of the
disputed product”
- “It fails to give any reason for the judgment it reaches.”
- “It introduces information unrelated to its conclusion as evidence in
support of that conclusion”
Internal
contradiction
Example: “Everyone should join our country club. After all, it’s an
exclusive group that links many of the influential members of the
community.”
- “bases a conclusion on claims that are inconsistent with each other”
- “the author makes incompatible assumptions”
- “introduce information that actually contradicts the conclusion”
- “assumes something that it later denies, resulting in a
contradiction”
Appeal Fallacies
Appeal to authority
The flaw in this form of reasoning is that the authority may not
have relevant knowledge or all the information regarding a
situation, or there may a difference of opinion among experts as
to what is true in the case. Example: use of a neurologist as an
authority figure in an area of dentistry
- “the judgement of experts is applied to a matter in which their
expertise is irrelevant”
- “the argument inappropriately appeals to the authority of the
mayor”
- “it relies on the judgment of experts in a matter to which their
expertise is irrelevant”
- “accepts a claim on mere authority, without requiring sufficient
justification”
COMMON ERRORS OF REASONING – 4
Appeal to
numbers/percentage
s/majority/popular
opinion
This error states that a position is true because the majority
believes it to be true
- “a claim is inferred to be false merely because a majority of people
believe it to be false”
- “the argument, instead of providing adequate reasons in support of
its conclusion, makes an appeal to popular opinion”
Appeal to Emotion An Appeal to Emotion occurs when emotions or emotionally-
charged language is used in an attempt to persuade the reader
Example: “Officer, please do not give me a ticket for speeding. In
the last month I’ve been fired from my job, kicked out of my
apartment, and my car broke down. I don’t deserve this!
- “attempts to persuade by making an emotional appeal”
- “uses emotive language in labelling the proposals”
- “the argument appeals to emotion rather than reason”
Survey Errors - Biased sample
- Improperly constructed/confusing survey questions/
misleading questions (how should the U.S. government
withdraw from the United Nations – presumes that the U.S.
government should withdraw, a course of action the
respondents might not agree with)
- Respondents give inaccurate responses
- “uses evidence drawn from a small sample that may well be
unrepresentative”
- “generalizes from an unrepresentative sample”
- “states a generalization based on a selection that is not
representative of the group about which the generalization is
supposed to hold true”
Exceptional
case/overgeneralizati
on
This error takes a small number of instances and treats those
instances as if they support a broad, sweeping conclusion
- “supports a universal claim on the basis of a single example”
- “The argument generalizes from too small a sample of cases”
- “bases a general claim on a few exceptional instances”
Errors of
Composition and
Division
Composition and division errors involve judgments made about
groups and parts of a group. An error of composition occurs when
the author attributes a characteristic of part of the group to the
group as a whole or to each member of the group
Example: “Every party I attend is fun and exciting. Therefore, my
life is fun and exciting”
An error of division occurs when the author attributes a
characteristic of the whole (or each member of the whole) to a
part of the group
Example: “The United States is the wealthiest country in the
world. Thus, every American is wealthy.”
- “assuming that because something is true of each of the parts of a
whole it is true of the whole itself”
- “improperly infers that each and every scientist has a certain
characteristic from the premise that most scientists have that
characteristic”
- “takes the view of one lawyer to represent the views of all
lawyers”
- “presumes, without providing justification, that what is true of a
whole must also be true of its constituent parts”
COMMON ERRORS OF REASONING – 5
Time Shift Errors The mistake involves assuming that conditions will remain constant over time, and that
what was the case in the past will be the case in the present or future
Example: “The company has always reimbursed me for meals when I’m on a business
trip, so they will certainly reimburse me for meals on this business trip.”
- “treats a claim about what is currently the case
as if it were a claim about what has been the
case for an extended period”
- “uncritically draws an inference from what has
been true in the past to what will be true in the
future”
Numbers and
Percentage Errors
Many errors in this category are committed when an author
improperly equates a percentage with a definite quantity, or when an author uses
quantity information to make a judgment about the percentage represented by that
quantity
- “the argument confuses the percentage of the
budget spent on a program with the overall
amount spent on that program”
Errors in the Use of
Evidence
Lack of evidence for a position is taken to prove that position is false
Example: “The White House has failed to offer any evidence that they have reached a
trade agreement with China. Therefore, no such agreement has been reached.” In the
example above the White House may have valid reasons for withholding information
about the trade agreement. The lack of confirming evidence does not undeniably
prove that a trade agreement has not been reached
- “treats failure to prove a claim as
constituting denial of that claim”
- “taking a lack of evidence for a claim as
evidence undermining that claim”
Lack of evidence against a position is taken to prove that position is true
This error is the opposite of the previous error. Just because no evidence disproving a
position has been introduced does not mean that the position is true
Example: “There has been no evidence given against the existence of God, so God
must exist.” The lack of evidence against a position does not undeniably prove a
position
- “treating the failure to establish that a
certain claim is false as equivalent to a
demonstration that the claim is true”
Some evidence against a position is taken to prove that position is false
The introduction of evidence against a position only weakens the position; it does not
necessarily prove the position false.
Example: “Some historians claim that a lengthy drought preceded the fall of the Aztec
empire. But we know from Aztec writings that in at least one year during the supposed
drought there was minor flooding. Thus, the claim that there was a lengthy drought
prior to the fall of the Aztec empire is false.”
- “it confuses undermining an argument in
support of a given conclusion with showing
that the conclusion itself is false”
Some evidence for a position is taken to prove that position is true
The introduction of evidence for a position only provides support for the position; it
does not prove the position to be undeniably true.
Example: “We know that the defendant was in the vicinity of the robbery when the
robbery occurred. Therefore, the defendant is guilty of the robbery.” As the example
proves, partial support for a position does not make the position invincible
- “the argument takes evidence showing
merely that its conclusion could be true to
constitute evidence showing that the
conclusion is in fact true”
Gmat verbal review
Gmat verbal review
Gmat verbal review
Gmat verbal review
Gmat verbal review
Gmat verbal review
Gmat verbal review

Más contenido relacionado

La actualidad más candente

La actualidad más candente (14)

Chapter 7 Grammar And Usage Slideshow Format
Chapter 7 Grammar And Usage Slideshow FormatChapter 7 Grammar And Usage Slideshow Format
Chapter 7 Grammar And Usage Slideshow Format
 
Grammar for Matric & Intermediate by Muhammad Azam
Grammar for Matric & Intermediate by Muhammad Azam Grammar for Matric & Intermediate by Muhammad Azam
Grammar for Matric & Intermediate by Muhammad Azam
 
Subject verb agreement exercise answers
Subject verb agreement exercise answersSubject verb agreement exercise answers
Subject verb agreement exercise answers
 
Academic writing skills
Academic writing skillsAcademic writing skills
Academic writing skills
 
Subject – verb agreement
Subject – verb agreementSubject – verb agreement
Subject – verb agreement
 
Grammar Review Mnemonic
Grammar Review MnemonicGrammar Review Mnemonic
Grammar Review Mnemonic
 
Subject verb agreement 1
Subject verb agreement 1Subject verb agreement 1
Subject verb agreement 1
 
Parts Of Speech
Parts Of SpeechParts Of Speech
Parts Of Speech
 
Grammar: Subject-Verb Agreement
Grammar: Subject-Verb AgreementGrammar: Subject-Verb Agreement
Grammar: Subject-Verb Agreement
 
Synonym, Antonym, and Hyponym
Synonym, Antonym, and HyponymSynonym, Antonym, and Hyponym
Synonym, Antonym, and Hyponym
 
Fun with Parts of Speech.
Fun with Parts of Speech.Fun with Parts of Speech.
Fun with Parts of Speech.
 
The parts of speech
The parts of speechThe parts of speech
The parts of speech
 
Grammar Review
Grammar ReviewGrammar Review
Grammar Review
 
Linkers
LinkersLinkers
Linkers
 

Similar a Gmat verbal review

PART-OF-SPEECH_CCA.pptx
PART-OF-SPEECH_CCA.pptxPART-OF-SPEECH_CCA.pptx
PART-OF-SPEECH_CCA.pptxSpyKids2
 
Use and form of adverb
Use and form of adverbUse and form of adverb
Use and form of adverbErikaElizath
 
Use and-form-of-adverb
Use and-form-of-adverbUse and-form-of-adverb
Use and-form-of-adverbLiset Ramirez
 
Use and form of adverb
Use and form of adverbUse and form of adverb
Use and form of adverbErikaElizath
 
Communication eng 101
Communication eng 101Communication eng 101
Communication eng 101RyanCapuchino
 
Communication eng 101
Communication eng 101Communication eng 101
Communication eng 101RyanCapuchino
 
Morphological processes
Morphological processesMorphological processes
Morphological processesLuqman Hakim
 
Writing Elements A Quick Guide to Grammar and Usag.docx
Writing Elements A Quick Guide to Grammar and Usag.docxWriting Elements A Quick Guide to Grammar and Usag.docx
Writing Elements A Quick Guide to Grammar and Usag.docxericbrooks84875
 
Eight-Parts-of-Speech.pptx
Eight-Parts-of-Speech.pptxEight-Parts-of-Speech.pptx
Eight-Parts-of-Speech.pptxMaylanieLamarca1
 
Learn English grammar
Learn English grammarLearn English grammar
Learn English grammarNAUMAN NAEEM
 
Subject-Verb Agreement - St Joseph School.pptx
Subject-Verb Agreement - St Joseph School.pptxSubject-Verb Agreement - St Joseph School.pptx
Subject-Verb Agreement - St Joseph School.pptxNniceRythm
 
Parts of speech review[1]
Parts of speech review[1]Parts of speech review[1]
Parts of speech review[1]Wendy Scruggs
 
Pronoun Agreement & Reference
Pronoun Agreement & ReferencePronoun Agreement & Reference
Pronoun Agreement & Referenceprofessorgriffith
 

Similar a Gmat verbal review (20)

PART-OF-SPEECH_CCA.pptx
PART-OF-SPEECH_CCA.pptxPART-OF-SPEECH_CCA.pptx
PART-OF-SPEECH_CCA.pptx
 
Grammar Session.pptx
Grammar Session.pptxGrammar Session.pptx
Grammar Session.pptx
 
Parts of speech
Parts of speechParts of speech
Parts of speech
 
Use and form of adverb
Use and form of adverbUse and form of adverb
Use and form of adverb
 
Use and-form-of-adverb
Use and-form-of-adverbUse and-form-of-adverb
Use and-form-of-adverb
 
Use and form of adverb
Use and form of adverbUse and form of adverb
Use and form of adverb
 
ADVERBS DOCUMENT
ADVERBS DOCUMENTADVERBS DOCUMENT
ADVERBS DOCUMENT
 
Eight Parts of Speech.pdf
Eight Parts of Speech.pdfEight Parts of Speech.pdf
Eight Parts of Speech.pdf
 
Communication eng 101
Communication eng 101Communication eng 101
Communication eng 101
 
Communication eng 101
Communication eng 101Communication eng 101
Communication eng 101
 
What is grammar
What is grammarWhat is grammar
What is grammar
 
Morphological processes
Morphological processesMorphological processes
Morphological processes
 
Writing Elements A Quick Guide to Grammar and Usag.docx
Writing Elements A Quick Guide to Grammar and Usag.docxWriting Elements A Quick Guide to Grammar and Usag.docx
Writing Elements A Quick Guide to Grammar and Usag.docx
 
Eight-Parts-of-Speech.pptx
Eight-Parts-of-Speech.pptxEight-Parts-of-Speech.pptx
Eight-Parts-of-Speech.pptx
 
Learn English grammar
Learn English grammarLearn English grammar
Learn English grammar
 
Vocabulary
VocabularyVocabulary
Vocabulary
 
syntax
 syntax syntax
syntax
 
Subject-Verb Agreement - St Joseph School.pptx
Subject-Verb Agreement - St Joseph School.pptxSubject-Verb Agreement - St Joseph School.pptx
Subject-Verb Agreement - St Joseph School.pptx
 
Parts of speech review[1]
Parts of speech review[1]Parts of speech review[1]
Parts of speech review[1]
 
Pronoun Agreement & Reference
Pronoun Agreement & ReferencePronoun Agreement & Reference
Pronoun Agreement & Reference
 

Último

Dum Dum ( Call Girls ) Kolkata ✔ 6297143586 ✔ Hot Model With Sexy Bhabi Ready...
Dum Dum ( Call Girls ) Kolkata ✔ 6297143586 ✔ Hot Model With Sexy Bhabi Ready...Dum Dum ( Call Girls ) Kolkata ✔ 6297143586 ✔ Hot Model With Sexy Bhabi Ready...
Dum Dum ( Call Girls ) Kolkata ✔ 6297143586 ✔ Hot Model With Sexy Bhabi Ready...ritikasharma
 
(TOP CLASS) Call Girls In Nungambakkam Phone 7427069034 Call Girls Model With...
(TOP CLASS) Call Girls In Nungambakkam Phone 7427069034 Call Girls Model With...(TOP CLASS) Call Girls In Nungambakkam Phone 7427069034 Call Girls Model With...
(TOP CLASS) Call Girls In Nungambakkam Phone 7427069034 Call Girls Model With... Shivani Pandey
 
Navi Mumbai Call Girls -📞9833754194-Call Girls Number Vashi-Nerul Call Girls ...
Navi Mumbai Call Girls -📞9833754194-Call Girls Number Vashi-Nerul Call Girls ...Navi Mumbai Call Girls -📞9833754194-Call Girls Number Vashi-Nerul Call Girls ...
Navi Mumbai Call Girls -📞9833754194-Call Girls Number Vashi-Nerul Call Girls ...priyasharma62062
 
College Call Girls Pune 8617697112 Short 1500 Night 6000 Best call girls Service
College Call Girls Pune 8617697112 Short 1500 Night 6000 Best call girls ServiceCollege Call Girls Pune 8617697112 Short 1500 Night 6000 Best call girls Service
College Call Girls Pune 8617697112 Short 1500 Night 6000 Best call girls ServiceNitya salvi
 
Verified Trusted Call Girls Tambaram Chennai ✔✔7427069034 Independent Chenna...
Verified Trusted Call Girls Tambaram Chennai ✔✔7427069034  Independent Chenna...Verified Trusted Call Girls Tambaram Chennai ✔✔7427069034  Independent Chenna...
Verified Trusted Call Girls Tambaram Chennai ✔✔7427069034 Independent Chenna... Shivani Pandey
 
📞 Contact Number 8617370543VIP Rajsamand Call Girls
📞 Contact Number 8617370543VIP Rajsamand Call Girls📞 Contact Number 8617370543VIP Rajsamand Call Girls
📞 Contact Number 8617370543VIP Rajsamand Call GirlsNitya salvi
 
Kanpur call girls 📞 8617697112 At Low Cost Cash Payment Booking
Kanpur call girls 📞 8617697112 At Low Cost Cash Payment BookingKanpur call girls 📞 8617697112 At Low Cost Cash Payment Booking
Kanpur call girls 📞 8617697112 At Low Cost Cash Payment BookingNitya salvi
 
Mumbai ] Call Girls Service Mumbai ₹7.5k Pick Up & Drop With Cash Payment 983...
Mumbai ] Call Girls Service Mumbai ₹7.5k Pick Up & Drop With Cash Payment 983...Mumbai ] Call Girls Service Mumbai ₹7.5k Pick Up & Drop With Cash Payment 983...
Mumbai ] Call Girls Service Mumbai ₹7.5k Pick Up & Drop With Cash Payment 983...hotbabesbook
 
Ranikhet call girls 📞 8617697112 At Low Cost Cash Payment Booking
Ranikhet call girls 📞 8617697112 At Low Cost Cash Payment BookingRanikhet call girls 📞 8617697112 At Low Cost Cash Payment Booking
Ranikhet call girls 📞 8617697112 At Low Cost Cash Payment BookingNitya salvi
 
Navsari Escorts Service ☎️ 6378878445 ( Sakshi Sinha ) High Profile Call Girl...
Navsari Escorts Service ☎️ 6378878445 ( Sakshi Sinha ) High Profile Call Girl...Navsari Escorts Service ☎️ 6378878445 ( Sakshi Sinha ) High Profile Call Girl...
Navsari Escorts Service ☎️ 6378878445 ( Sakshi Sinha ) High Profile Call Girl...mriyagarg453
 
Sonagachi ( Call Girls ) Kolkata ✔ 6297143586 ✔ Hot Model With Sexy Bhabi Rea...
Sonagachi ( Call Girls ) Kolkata ✔ 6297143586 ✔ Hot Model With Sexy Bhabi Rea...Sonagachi ( Call Girls ) Kolkata ✔ 6297143586 ✔ Hot Model With Sexy Bhabi Rea...
Sonagachi ( Call Girls ) Kolkata ✔ 6297143586 ✔ Hot Model With Sexy Bhabi Rea...rahim quresi
 
Call Girls Panaji Just Call 8617370543 Top Class Call Girl Service Available
Call Girls Panaji Just Call 8617370543 Top Class Call Girl Service AvailableCall Girls Panaji Just Call 8617370543 Top Class Call Girl Service Available
Call Girls Panaji Just Call 8617370543 Top Class Call Girl Service AvailableNitya salvi
 
𓀤Call On 6297143586 𓀤 Sonagachi Call Girls In All Kolkata 24/7 Provide Call W...
𓀤Call On 6297143586 𓀤 Sonagachi Call Girls In All Kolkata 24/7 Provide Call W...𓀤Call On 6297143586 𓀤 Sonagachi Call Girls In All Kolkata 24/7 Provide Call W...
𓀤Call On 6297143586 𓀤 Sonagachi Call Girls In All Kolkata 24/7 Provide Call W...rahim quresi
 
Zirakpur Call Girls👧 Book Now📱8146719683 📞👉Mohali Call Girl Service No Advanc...
Zirakpur Call Girls👧 Book Now📱8146719683 📞👉Mohali Call Girl Service No Advanc...Zirakpur Call Girls👧 Book Now📱8146719683 📞👉Mohali Call Girl Service No Advanc...
Zirakpur Call Girls👧 Book Now📱8146719683 📞👉Mohali Call Girl Service No Advanc...rajveermohali2022
 
Low Rate Call Girls Dhakuria (8005736733) 100% GENUINE ESCORT SERVICE & HOTEL...
Low Rate Call Girls Dhakuria (8005736733) 100% GENUINE ESCORT SERVICE & HOTEL...Low Rate Call Girls Dhakuria (8005736733) 100% GENUINE ESCORT SERVICE & HOTEL...
Low Rate Call Girls Dhakuria (8005736733) 100% GENUINE ESCORT SERVICE & HOTEL... Shivani Pandey
 
Hire 💕 8617697112 North Sikkim Call Girls Service Call Girls Agency
Hire 💕 8617697112 North Sikkim Call Girls Service Call Girls AgencyHire 💕 8617697112 North Sikkim Call Girls Service Call Girls Agency
Hire 💕 8617697112 North Sikkim Call Girls Service Call Girls AgencyNitya salvi
 
Model VVIP Call Girls In Porur 👉 Chennai 🍬 7427069034 Escort Service & Hotel ...
Model VVIP Call Girls In Porur 👉 Chennai 🍬 7427069034 Escort Service & Hotel ...Model VVIP Call Girls In Porur 👉 Chennai 🍬 7427069034 Escort Service & Hotel ...
Model VVIP Call Girls In Porur 👉 Chennai 🍬 7427069034 Escort Service & Hotel ...hotbabesbook
 
Bhimtal ❤CALL GIRL 8617697112 ❤CALL GIRLS IN Bhimtal ESCORT SERVICE❤CALL GIRL
Bhimtal ❤CALL GIRL 8617697112 ❤CALL GIRLS IN Bhimtal ESCORT SERVICE❤CALL GIRLBhimtal ❤CALL GIRL 8617697112 ❤CALL GIRLS IN Bhimtal ESCORT SERVICE❤CALL GIRL
Bhimtal ❤CALL GIRL 8617697112 ❤CALL GIRLS IN Bhimtal ESCORT SERVICE❤CALL GIRLNitya salvi
 
Call Girls Bellandur ☎ 7737669865☎ Book Your One night Stand (Bangalore)
Call Girls Bellandur ☎ 7737669865☎ Book Your One night Stand (Bangalore)Call Girls Bellandur ☎ 7737669865☎ Book Your One night Stand (Bangalore)
Call Girls Bellandur ☎ 7737669865☎ Book Your One night Stand (Bangalore)amitlee9823
 

Último (20)

Dum Dum ( Call Girls ) Kolkata ✔ 6297143586 ✔ Hot Model With Sexy Bhabi Ready...
Dum Dum ( Call Girls ) Kolkata ✔ 6297143586 ✔ Hot Model With Sexy Bhabi Ready...Dum Dum ( Call Girls ) Kolkata ✔ 6297143586 ✔ Hot Model With Sexy Bhabi Ready...
Dum Dum ( Call Girls ) Kolkata ✔ 6297143586 ✔ Hot Model With Sexy Bhabi Ready...
 
(TOP CLASS) Call Girls In Nungambakkam Phone 7427069034 Call Girls Model With...
(TOP CLASS) Call Girls In Nungambakkam Phone 7427069034 Call Girls Model With...(TOP CLASS) Call Girls In Nungambakkam Phone 7427069034 Call Girls Model With...
(TOP CLASS) Call Girls In Nungambakkam Phone 7427069034 Call Girls Model With...
 
Navi Mumbai Call Girls -📞9833754194-Call Girls Number Vashi-Nerul Call Girls ...
Navi Mumbai Call Girls -📞9833754194-Call Girls Number Vashi-Nerul Call Girls ...Navi Mumbai Call Girls -📞9833754194-Call Girls Number Vashi-Nerul Call Girls ...
Navi Mumbai Call Girls -📞9833754194-Call Girls Number Vashi-Nerul Call Girls ...
 
College Call Girls Pune 8617697112 Short 1500 Night 6000 Best call girls Service
College Call Girls Pune 8617697112 Short 1500 Night 6000 Best call girls ServiceCollege Call Girls Pune 8617697112 Short 1500 Night 6000 Best call girls Service
College Call Girls Pune 8617697112 Short 1500 Night 6000 Best call girls Service
 
Verified Trusted Call Girls Tambaram Chennai ✔✔7427069034 Independent Chenna...
Verified Trusted Call Girls Tambaram Chennai ✔✔7427069034  Independent Chenna...Verified Trusted Call Girls Tambaram Chennai ✔✔7427069034  Independent Chenna...
Verified Trusted Call Girls Tambaram Chennai ✔✔7427069034 Independent Chenna...
 
📞 Contact Number 8617370543VIP Rajsamand Call Girls
📞 Contact Number 8617370543VIP Rajsamand Call Girls📞 Contact Number 8617370543VIP Rajsamand Call Girls
📞 Contact Number 8617370543VIP Rajsamand Call Girls
 
Kanpur call girls 📞 8617697112 At Low Cost Cash Payment Booking
Kanpur call girls 📞 8617697112 At Low Cost Cash Payment BookingKanpur call girls 📞 8617697112 At Low Cost Cash Payment Booking
Kanpur call girls 📞 8617697112 At Low Cost Cash Payment Booking
 
Mumbai ] Call Girls Service Mumbai ₹7.5k Pick Up & Drop With Cash Payment 983...
Mumbai ] Call Girls Service Mumbai ₹7.5k Pick Up & Drop With Cash Payment 983...Mumbai ] Call Girls Service Mumbai ₹7.5k Pick Up & Drop With Cash Payment 983...
Mumbai ] Call Girls Service Mumbai ₹7.5k Pick Up & Drop With Cash Payment 983...
 
Ranikhet call girls 📞 8617697112 At Low Cost Cash Payment Booking
Ranikhet call girls 📞 8617697112 At Low Cost Cash Payment BookingRanikhet call girls 📞 8617697112 At Low Cost Cash Payment Booking
Ranikhet call girls 📞 8617697112 At Low Cost Cash Payment Booking
 
Navsari Escorts Service ☎️ 6378878445 ( Sakshi Sinha ) High Profile Call Girl...
Navsari Escorts Service ☎️ 6378878445 ( Sakshi Sinha ) High Profile Call Girl...Navsari Escorts Service ☎️ 6378878445 ( Sakshi Sinha ) High Profile Call Girl...
Navsari Escorts Service ☎️ 6378878445 ( Sakshi Sinha ) High Profile Call Girl...
 
Sonagachi ( Call Girls ) Kolkata ✔ 6297143586 ✔ Hot Model With Sexy Bhabi Rea...
Sonagachi ( Call Girls ) Kolkata ✔ 6297143586 ✔ Hot Model With Sexy Bhabi Rea...Sonagachi ( Call Girls ) Kolkata ✔ 6297143586 ✔ Hot Model With Sexy Bhabi Rea...
Sonagachi ( Call Girls ) Kolkata ✔ 6297143586 ✔ Hot Model With Sexy Bhabi Rea...
 
CHEAP Call Girls in Malviya Nagar, (-DELHI )🔝 9953056974🔝(=)/CALL GIRLS SERVICE
CHEAP Call Girls in  Malviya Nagar, (-DELHI )🔝 9953056974🔝(=)/CALL GIRLS SERVICECHEAP Call Girls in  Malviya Nagar, (-DELHI )🔝 9953056974🔝(=)/CALL GIRLS SERVICE
CHEAP Call Girls in Malviya Nagar, (-DELHI )🔝 9953056974🔝(=)/CALL GIRLS SERVICE
 
Call Girls Panaji Just Call 8617370543 Top Class Call Girl Service Available
Call Girls Panaji Just Call 8617370543 Top Class Call Girl Service AvailableCall Girls Panaji Just Call 8617370543 Top Class Call Girl Service Available
Call Girls Panaji Just Call 8617370543 Top Class Call Girl Service Available
 
𓀤Call On 6297143586 𓀤 Sonagachi Call Girls In All Kolkata 24/7 Provide Call W...
𓀤Call On 6297143586 𓀤 Sonagachi Call Girls In All Kolkata 24/7 Provide Call W...𓀤Call On 6297143586 𓀤 Sonagachi Call Girls In All Kolkata 24/7 Provide Call W...
𓀤Call On 6297143586 𓀤 Sonagachi Call Girls In All Kolkata 24/7 Provide Call W...
 
Zirakpur Call Girls👧 Book Now📱8146719683 📞👉Mohali Call Girl Service No Advanc...
Zirakpur Call Girls👧 Book Now📱8146719683 📞👉Mohali Call Girl Service No Advanc...Zirakpur Call Girls👧 Book Now📱8146719683 📞👉Mohali Call Girl Service No Advanc...
Zirakpur Call Girls👧 Book Now📱8146719683 📞👉Mohali Call Girl Service No Advanc...
 
Low Rate Call Girls Dhakuria (8005736733) 100% GENUINE ESCORT SERVICE & HOTEL...
Low Rate Call Girls Dhakuria (8005736733) 100% GENUINE ESCORT SERVICE & HOTEL...Low Rate Call Girls Dhakuria (8005736733) 100% GENUINE ESCORT SERVICE & HOTEL...
Low Rate Call Girls Dhakuria (8005736733) 100% GENUINE ESCORT SERVICE & HOTEL...
 
Hire 💕 8617697112 North Sikkim Call Girls Service Call Girls Agency
Hire 💕 8617697112 North Sikkim Call Girls Service Call Girls AgencyHire 💕 8617697112 North Sikkim Call Girls Service Call Girls Agency
Hire 💕 8617697112 North Sikkim Call Girls Service Call Girls Agency
 
Model VVIP Call Girls In Porur 👉 Chennai 🍬 7427069034 Escort Service & Hotel ...
Model VVIP Call Girls In Porur 👉 Chennai 🍬 7427069034 Escort Service & Hotel ...Model VVIP Call Girls In Porur 👉 Chennai 🍬 7427069034 Escort Service & Hotel ...
Model VVIP Call Girls In Porur 👉 Chennai 🍬 7427069034 Escort Service & Hotel ...
 
Bhimtal ❤CALL GIRL 8617697112 ❤CALL GIRLS IN Bhimtal ESCORT SERVICE❤CALL GIRL
Bhimtal ❤CALL GIRL 8617697112 ❤CALL GIRLS IN Bhimtal ESCORT SERVICE❤CALL GIRLBhimtal ❤CALL GIRL 8617697112 ❤CALL GIRLS IN Bhimtal ESCORT SERVICE❤CALL GIRL
Bhimtal ❤CALL GIRL 8617697112 ❤CALL GIRLS IN Bhimtal ESCORT SERVICE❤CALL GIRL
 
Call Girls Bellandur ☎ 7737669865☎ Book Your One night Stand (Bangalore)
Call Girls Bellandur ☎ 7737669865☎ Book Your One night Stand (Bangalore)Call Girls Bellandur ☎ 7737669865☎ Book Your One night Stand (Bangalore)
Call Girls Bellandur ☎ 7737669865☎ Book Your One night Stand (Bangalore)
 

Gmat verbal review

  • 3. SENTENCE STRUCTURE PHRASE CLAUSE SENTENCE A clause is a group of related words that must have a subject and a verb. Clauses may be classified into two categories: • An independent clause is one which contains a subject and a verb, and expresses a complete thought. For example, Jane took the train • A dependent clause is one which contains a subject and a verb, but does not express a complete thought, and cannot stand for itself. For example, after Jane took the train. The clause raises a question – what happened after Jane took the train? Example: Barry ran, as he wanted to reach on time Independent clause: Barry ran Dependent clause: as he wanted to reach on time Group of related words which does not express a complete thought, and where the subject, verb or both, are missing • Prepositional phrase: at the beach, near the stove, at this time • Verbal Phase: building a large dam, to fly in a plane, signed the legal document Examples: - The sun rises in the east - Humpty Dumpty sat on a wall - She wore a hat with blue trimming - The accident on the bridge was not serious - The girl with red hair is an artist • Must have a subject • Must have a verb • Must make common-sense/form a complete thought COMMON ERRORS Common splice This occurs when two independent clauses in a sentence are combined using a comma instead of using a semi-colon or an appropriate conjunction Incorrect: Tom read the novel, his friend saw the movie Correct: Tom read the novel, but his friend saw the movie Correct: Tom read the novel; his friend saw the movie
  • 4. PARTS OF SPEECH Part of speech Definition Example Noun Concrete nouns: person, animal, thing, place Abstract nouns: You cannot see them, hear them, smell them, taste them, or feel them (physically) Concrete nouns: teacher, airplane, dog Abstract nouns: curiosity, love, happiness Pronoun Word that takes the place of a noun Example: I, you, he, she, it, we, them, me, you, him, her, it, us, them, mine, yours, his, hers, its, ours, theirs, this, that, which, who Verb Verbs express action – something that a person, animal, force of nature, or thing can DO. For instance, sneezing, laughing, winking State of being – linking verbs connect the subject of a verb to additional information about the subject. Some examples include, any form of the verb be – am, were, has been, are being, might have been – become, and seem A three-mile run seems like a marathon during a hot, humid July afternoon During bad storms, trailer parks are often magnets for tornadoes Adjective A word that modifies a NOUN or a PRONOUN by describing, identifying, or quantifying words - What kind? - Which one? - How much? Smith is a famous explorer Sidney Sheldon has written thirty novels Adverb A word that modifies everything but nouns and pronouns – modifies ADJECTIVES, VERBS AND OTHER ADVERBS. Most adverbs end in –ly. A word is an adverb if it answers - How? - When? - Where? - Why? We’ll use the new software program tomorrow He climbed the ladder slowly Conjunction Connecting words: and, but, or, nor, for, yet, so, although, because, since, unless Jack and Jill went up the hill Preposition Links nouns, phrases and pronouns to other words in the sentence The book is on the table
  • 5. THREE KEY CONCEPTS IN SENTENCE CORRECTION CONCISION Refers to brevity – is the sentence written as economically as possible? Avoid redundancy - if a word can be removed without altering the meaning of the sentence, it is redundant and should be eliminated Example: • Past experience reveals that cancer patients rarely ever exhibit the exact same symptoms • In the example above, the word “past” is implicit in “experience” and can be eliminated. Similarly the words “ever” and “exact” are only used for emphasis (a practice the GMAT does not condone) • Correct: Experience reveals that cancer patients rarely exhibit the same symptoms A common redundancy trap is the use of words with the same meaning Example: • The value of the stock rose by a 10% increase • Since rose and increase both imply growth, only one is needed to convey the correct meaning “Being” almost always signals redundancy and should be avoided wherever possible CORRECTNESS Subject – verb agreement; Verb tense; Voice & mood; Pronouns; Modifiers; Parallelism; Comparisons; Idioms CLARITY Refers to intelligibility – is the meaning of the sentence obvious and unambiguous? If an answer choice alters the original meaning (or intent) of the sentence, it is incorrect. Most instances of altered meaning fall into four broad categories: 1. Word placement 2. Known vs. Unknown i. The original sentence is certain about an outcome but the answer choices indicate uncertainty (or vice versa) ii. The original sentence discusses a hypothetical situation but the answer choices present it as an actual situation (or vice versa) 3. Multiple meanings 4. Such as vs. Like
  • 6. THREE KEY CONCEPTS IN SENTENCE CORRECTION – CLARITY CONTINUED Category Description Example Word Placement Be aware of words that move from one position to the other; the placement of a single word can alter the meaning of the sentence All the children are covered with mud The children are all covered with mud In these sentences, the placement of all shifts the focus from the number of children covered with mud to the extent to which the children are covered by it Known vs. Unknown The original sentence is certain about an outcome but the answer choices indicate uncertainty (or vice versa) Certain: The court ruled that the plaintiff must pay full damages Uncertain: The court ruled that the plaintiff should pay full damages The original sentence discusses a hypothetical situation but the answer choices present it as an actual situation (or vice versa) Actual: If Newton and Liebniz met, they discussed mathematics Hypothetical: If Newton and Liebniz met, they would discuss mathematics Multiple meanings Some words can be read in more than one way, altering the meaning of the sentence according to the given interpretation Sentence # 1: The light fabric makes the shirt easy to fold Sentence # 2: The shirt is easy to fold, and very light The first sentence makes it clear that in this context the shirt is “not heavy”. However, in the second sentence, light could mean either “pale” or “not heavy”. The context does not establish the meaning clearly “Such as” vs. “Like” “Such as” is used to indicate examples; “Like” is used to indicate similarity Animals such as zebras and lions live on the Serengeti Plain Animals like zebras and lions live on the Serengeti Plain The first sentence indicates that lions and zebras are specific types of animals that live on the Serengeti Plain. The second sentence indicates that animals similar to lions and zebras live on the Serengeti Plain, but it is not clear whether lions and zebras themselves actually do
  • 7. SUBJECT – VERB AGREEMENT The basic rule states that a singular subject takes a singular verb, while a plural subject takes a plural verb First determine whether the subject in the sentence or the clause is singular or plural • To find the subject, always find the verb first, and then ask who or what performed the verb • It is important to note that verbs do not form their plurals by adding an s as nouns do. In order to determine which verb is singular and which one is plural, think of which verb you would use with he or she and which verb you would use with they • If a verb follows to, it is called an infinitive phrase and is not the main verb. You will find the main verb either before or after the infinitive phrase Example: The efforts to get her elected succeeded ELIMINATE THE MIDDLEMAN The intervening phase between the subject and the verb should be eliminated so that the true subject becomes clear • “of” and “for” are two common middlemen • Prepositional phrases Examples: The houses of that rich man (contain/contains) very expensive furniture The subject – houses – is plural, and hence requires the plural verb form contain • The discovery of new lands (was/were) important to the expansion of the British empire • The building of tall skyscrapers (has/have) increased in the past few years • The actions of my friend (is/are) not very wise • The book I bought for my students (tell/tells) the story of a Russian immigrant’s experience in the United States COLLECTIVE NOUNS ARE GENERALLY SINGULAR A collective noun is a noun that looks singular (it usually does not end with an “s”) but refers to a group of people Collective nouns are singular when they act in a collective fashion or represent a group. Examples include team, family, majority, administration, army, audience, class, crowd, facult y, audience, and committee Collective nouns are plural when the members of the collective body act as individuals • The team are always fighting amongst themselves • While 'team' is often used as a singular collective noun, in this case, the sentence describes the fighting that occurs between the individual members of the team. "Team" therefore refers to several individual members, and requires a plural verb, "are," as a result
  • 8. SUBJECT – VERB AGREEMENT AND vs. ADDITIVES The word “and” can unite two or more singular subjects, forming a compound plural subject • John and his friends ARE going to the beach When the two singular noun subjects connected by “and” refer to the same person or thing, the singular verb is used • Bread and butter is his staple diet • The recently appointed manager and coach of the team has emphasized the need for fitness Additive phrases such as: along with, in addition to, as well as, accompanied by, together with, including, do not form compound plural subjects • Correct: John, along with his friends, IS going to the beach • Incorrect: Frank, accompanied by his students, were at the studio • Correct: Frank, accompanied by his students, was at the studio INFINITE PRONOUNS ARE SINGULAR An infinite pronoun is one which is not specific about the thing to which it refers All pronouns which end in –body, – one, or –thing fall under this category: - Anyone, anything, anybody - Everyone, everybody, everything - Each, every - Whatever, whoever, no one, nobody, nothing - Either and neither (may require a plural verb form when paired with or/nor) • Neither of the rosebushes is as pretty as last year • Either of us is capable of doing the work • Neither John nor his friends are going to the beach There are 5 infinite pronouns which can be either singular or plural depending on the context of the sentence: Some, Any, None, All, Most (SANAM) VERB FORM IN EITHER/OR , NEITHER/NOR DEPENDS ON THE SUBJECT NEAREST TO THE VERB In these sentences, there are two subjects. If one subject is singular, and the other is plural, find the subject nearest to the verb, and ensure that the verb agrees in number with this subject • Neither Joe nor his friends ARE going to the beach • Neither his friends nor Joe IS going to the beach EACH/EVERY – GENERALLY SINGULAR SENSATIONS When each/every is the subject, the verb form is singular • Each of these shirts IS pretty When the subject is preceded by each/every, the verb form is singular • Every dog and cat HAS paws When each/every follows a subject, it has no bearing on the verb form • They each ARE great tennis players • The apartments each HAVE their own private entrance
  • 9. SUBJECT – VERB AGREEMENT NUMERICAL WORDS AND PHRASES The phrase, “the number of” always takes a singular verb form The phrase, “a number of” always takes a plural verb form Numbers greater than 1 are plural • The number of students in this class IS quite large • A number of students in this class ARE hardworking • TWO out of every three dog owners in the U.S. also OWN a cat PORTIONS With words that indicate portions—percent, fraction, part, majority, some, all, none, remainder, and so forth —look at the noun in your of phrase (object of the preposition) to determine whether to use a singular or plural verb If one means the many individual parts of the totality, then use a plural verb form Example: The majority of the students in the class ARE hard working If one means the totality itself, then use a singular verb form Example: The student majority IS opposed to the death penalty • Half of the dresses ARE dirty • Half of the sugar IS over • Two-thirds of the work IS complete • The majority of Congress has voted for the bill “ONE OF” The phrases one of and one of the take a singular verb • One of you is telling the truth • One of the editors wants a rewrite Plural - The phrases one of those who and one of the things/factors Singular - When the word “the only” precedes these phrases • The comma splice is one of those errors that always slip past me • One of the things that drive me nuts is subject-verb agreement • Meg is the only one who knows how to paddle a canoe • Ted is not the only one of my nephews who has a vivid imagination MISCELLANEOUS • “Both... and” is the ONLY pair that always results in a plural subject • Gerunds (-ing form of the verb and functioning as nouns) are singular subjects • When in doubt, think singular • When a sentence has both a positive and a negative subject, the verb agrees with the positive subject. For example: Low prices and not quality determine many purchases; Peace of mind not riches is what makes a person happy
  • 10. VERB TENSE VERB TENSE indicates when an action takes place. The basic tenses are present, past and future Unless actions do not take place at the same time, keep all verb tenses in a given sentence the same Example • She walked to school in the morning and ran home in the afternoon • She walks to school in the morning and runs home in the afternoon • She will walk to school in the morning and run home in the afternoon (Note: run is understood as will run) • Typically, –ing forms are sometimes used as junk answers on the GMAT; there will often be a better alternative • Tip: Pick one event as the base action  determine when the other actions occurred in relation to that event  determine the proper verb form • “Will” indicates future tense; “would” indicates future from the past • “Might” is the past tense of “May” • Progressive tense indicates an ongoing action – in the past, present or future • The perfect progressive form indicates an action that is ongoing, but will be completed at some definite time • Infinitive verbs: “To” + verb. This is considered as the most basic form of the verb, or the building block of all other tenses. To form other tenses of the verb, you simply modify the infinitive form - Avoid sentences that insert a word (or words) between “to” and the verb (rarely tested, but almost always incorrect) IF & THEN CLAUSE IF Clause THEN clause Example Present will + base verb If she wins the lottery, she will give half the money to charity Past would/could + base verb If she won the lottery, she would give half the money to charity Future would/could + have + past participle If she had won the lottery, she would have given half the money to charity ** Conditional words “would” and “could” NEVER appear in the IF clause
  • 11. VERB TENSE - PRESENT TENSE Tense When to Use How to Use Simple present Generally expresses events, or situations that have existed in the past, exist now (in the present) and will probably exist in the future - I understand what you are saying General truths - Grass is green Present perfect If an event started in the past but continues (or remains true in) in the present / actions have not yet finished - We have lived in a little hut for three days - Our country has enforced strict immigration laws for thirty years When an event occurred at an indeterminate point in the past - They have known each other for the longest time - I have been to California If the time period has not finished - I have seen three movies this week Giving recent news - Michael has crashed his car again Has/Have + Past participle. Past participle of a regular verb is verb + -ed ending (example: walked, danced, jumped) Present perfect continuous Recent activity - She's been working hard recently Emphasis on the duration or length of an activity - Jack has been painting for 4 hours Recently finished activity with a present result - I've been working in the garden, that's why my hands are so dirty Present continuous If an event is occurring at this very moment / around now / near future (especially planned activities) - The kids are watching TV - We aren't working hard these days - Polly is coming for dinner tomorrow Am/is/are + verb-ing
  • 12. VERB TENSE - PAST TENSE Tense When to Use How to Use Simple past If an event started and finished at a particular point of time in the past - Molly and Rita embraced at the airport - George ate all his dinner Past perfect If more than one event in the sentence occurred at different times in the past, use the past perfect tense for the earlier action (FINISHED action) and simple past for the later action - The film had started by the time we arrived at the theatre - The teacher thought that Johnny had cheated on his exam Had + past participle Past perfect continuous Past perfect progressive is used to express CONTINUOUS activity up to a specific point of time in the past - They had been waiting for 2 hours before their friends finally arrived Past continuous Duration in the past - actions or situations that lasted for some time in the past, and whose duration time is unknown or unimportant - The dog was barking Interrupted actions in progress – when one action in progress is interrupted by another action in the past. We usually use when or while to link these two sentences. - I was talking with James when the telephone rang Actions in progress at the same time in the past - two or more activities happening at the same. We usually use when or while to link the two sentences - When Bob was painting windows, Mary was working in the kitchen
  • 13. VERB TENSE - FUTURE TENSE Tense When to Use How to Use Simple future Promises - I promise I will buy you this toy Unplanned actions (spontaneous decisions) - Don't worry! I will help you with this problem Predictions based on experience or intuition - It will rain in a moment Habits (obstinate insistence, usually habitual) - She will bite her lip if she is thinking or if she's nervous about something ** Remember, you should never use will to say what somebody has already arranged or decided to do in the future: Correct: Mike is moving to New York next week Incorrect: Mike will move to New York next week Future perfect The Future Perfect expresses the idea that something will occur before another action in the future. It can also show that something will happen before a specific time in the future - I will have been in London for six months by the time I leave - Sam is probably going to have completed the proposal by the time he leaves this afternoon Incorrect: By the time I write to Leo, he will probably move Correct: By the time I write to Leo, he will probably have moved Will have + past participle Am/is/are + going to have + past participle Future perfect continuous Emphasize the length of time or duration of an event occurring before and up to another event in the future - John will have been studying for 6 years by the time he finishes his exam Future continuous Future actions in progress - In an hour, I will be sitting in front of my TV Guesses about the present/future - He won't be coming any time soon. He is still at the office Polite questions about somebody's intention - Will you be going to the supermarket? I have something to buy Will + be + verb-ing
  • 14. MOODS SUBJUNCTIVE Verbs in subjunctive mood deal with events that are not necessarily true INDICATIVE Verbs in indicative mood deal with real events IF CLAUSE When the “if” clause represents something contrary to reality - In these cases, the verb is always “were”, regardless of the subject • If he were tall, he would be able to play basketball better “That” and “That” Parallelism is to be maintained • Incorrect: He said that he had studied well and he would score well • Correct: He said that he had studied well and that he would score well Hopes, dreams, desires and requests formed with the word “that” It is the desire of one person or body for another person or body to do something. It is unclear as to whether or not the second person or body will actually do what is asked Subjunctive is formed with “that + base form of the verb” and follows words such as advice, advisable, ask, arrange, better, demand, desire, instruct, instructions, intend, intentions, order, pray, prefer, propose, recommend, suggest, request, require, urge, urgent, vital etc. The base form of the verb is always plural • Correct: The parolee knew it was imperative that he find a job quickly (base form of the verb) • Incorrect: It is imperative that she must sign the permission slip (must is redundant) • Correct: It is imperative that she sign the permission slip
  • 15. VOICE ACTIVE VOICE PASSIVE VOICE The subject of the sentence has an action performed on it by someone or something else - The pizza was eaten by the hungry students Passive voice is formed with a form of the verb “to be”, followed by a participle. The person or people performing the action in the sentence almost always follow the verb Required when the non-underlined portion of the sentence contains the person or agent performing the action preceded by the word “by” • Incorrect: The shuttle launch seen around the world by people of all ages, all races and all religions (missing a verb, and therefore is a fragment) • Correct: The shuttle launch was seen around the world by people of all ages, all races and all religions SUBJECT PERFORMS THE ACTION - The hungry students ate the pizza Active voice is preferred to passive voice. Only transitive verbs (verbs that take direct objects) can be written in passive voice - The aliens arrived on Neptune in the 20th century
  • 16. PRONOUNS Subject Pronouns Object Pronouns Possessive Pronouns I Me My, mine You You Your, yours He Him His She Her Her, hers It It Its (not it’s) We Us Our, ours They Them Their, theirs Who Whom Whose Who vs. whom To simplify who/whom questions, try rearranging the sentence into a question, and then answer it Question: Who/m did Kate marry? Answer: Kate married him Since the pronoun used in the answer is "him," an object pronoun, the pronoun in the original sentence should also be an object pronoun: whom Example # 1 • Incorrect: Janice and ME went on a picnic together • Correct: Janice and I went on a picnic together • The pronoun is part of the subject, and hence, the correct form is I and not me Example # 2 • Incorrect: The picnic was attended by Janice and I • Correct: The picnic was attended by Janice and me • Picnic is the subject, while the pronoun is part of the object, thus the correct form is me and not I Example # 3 Marston was an early seventeenth century dramatist and it is likely that him and Shakespeare borrowed ideas from one another. • It is likely that him and Shakespeare borrowed ideas from one another • It is likely that they borrowed ideas from one another • It is likely that him and Shakespeare borrowed ideas from each other • It is likely that himself and Shakespeare borrowed ideas from one another • It is likely that he and Shakespeare borrowed ideas from one another Explanation: This question tests pronoun agreement. The pronoun ‘him’ in the original sentence replaces ‘Marston.’ It is in the wrong case. Instead of the objective case, the pronoun should be in the subjective case, since Marston is the subject of the sentence. Therefore (E), which uses the subjective pronoun ‘he,’ is correct. (B) is wrong because, though ‘they’ is subjective, it eliminates any meaning of Shakespeare from the sentence, making the pronoun ambiguous • Adding a pronoun in an answer choice more often than not makes the answer choice wrong. However, sometimes it is essential to introduce a pronoun to add clarity • The first step to solving a pronoun question is to identify all the pronouns • A relative pronoun (i.e., that, which, who etc.) must refer to the word immediately preceding it • A single pronoun can only refer to a noun, not a noun and a verb
  • 17. PRONOUNS – 1 Implication is not enough; there must be a stated antecedent for every pronoun Friendship was something that James always valued, so he disliked it when THEY talked about him behind his back He, him and his clearly refers to James. However, the pronoun THEY has no antecedent at all. One might think that THEY refers to friends, but the word friends is never mentioned in the sentence, only the word friendship is. Therefore, the sentence is incorrect A pronoun must refer to one and ONLY one antecedent (noun) After finding the antecedent, check whether the pronoun agrees in number ** The deadly four: it, its, they, their • Incorrect: Police work is very important as THEY help to enforce the laws of the state • Correct: Policemen are very important as THEY help to enforce the laws of the state • Correct: Police work is very important as IT is the backbone of the state Example # 1 • Incorrect: When the person calls, take down THEIR phone number • Correct: When the person calls, take down HIS phone number • Correct: When the people call, take down THEIR phone number Example # 2 • Incorrect: Everyone here will need THEIR own pencil • Correct: Everyone here will need HIS own pencil Pronoun must agree in number (singular/plural) with the antecedent
  • 18. PRONOUNS – 2 Possessive pronouns can refer back to possessive nouns. However, subject and object pronouns may NOT refer back to possessive nouns. Subject and object pronouns may only refer back to subject and object nouns • Incorrect: Jose’s room is so messy that his mother calls HIM a pig • Correct: Jose’s room is so messy that his mother calls Jose a pig POSSESSIVE POISON Never use one or one’s to refer back to any antecedent except one. Similar for you Example # 1 • Incorrect: A person should leave a light on in an empty house if one wants to give the impression that someone is at home • Correct: Incorrect: A person should leave a light on in an empty house if he or she wants to give the impression that someone is at home • Correct: One should leave a light on in an empty house if one wants to give the impression that someone is at home Example # 2 • Incorrect: One should have their teeth checked every six months • Correct: One should have one's teeth checked every six months • Correct: You should have your teeth checked every six months IMPERSONAL PRONOUNS – ONE / YOU
  • 19. MODIFIERS A modifier or a modifying phrase describes someone or something in the sentence A modifier must always be placed as close as possible to the word it's modifying ADJECTIVES An adjective describes ONLY a noun or pronoun and answers the questions: how many, which one, what kind? The following “sense” verbs (verbs that describe someone's sensation or feeling or perception) require adjective modifiers: - Sound - Look - Smell - Taste - Feel - Seem • The smart man acts quickly • The adjective smart modifies the noun man, while the adverb quickly modifies the verb acts • Incorrect: After she returned from the three-week vacation, she looked very well (looked is a sense word, well is a adverb) • Correct: After she returned from the three-week vacation, she looked very good (adjective) ADVERBS An adverb usually modifies a verb, but it can also describe an adjective, another adverb, a preposition or a phrase. Many adverbs are formed by adding –ly to the adjective • Incorrect: My friend is a real interesting person • Correct: My friend is a really interesting person • The adverb really must be used to modify the adjective interesting
  • 20. MODIFIERS – 1 MODIFIER ERRORS POSSESSIVE POSIONDANGLING MODIFIERS MISPLACED MODIFIERS When the modified noun is not present • Incorrect: Using the latest technology, the mechanical problem was identified quickly • The modifier, using the latest technology, is probably describing the technician who identified the problem. However, a technician never appears in the sentence • Correct: Using the latest technology, the engineer identified the problem quickly This occurs when the modified noun is not placed directly next to the modifying phrase. A modifying phrase must not be separated from the noun it modifies A modifying phrase that begins a sentence refers to the noun or pronoun immediately following the phrase Example # 1 • Incorrect: Upon leaving the register, the cashier handed the customer a receipt • The modifier, upon leaving the register seems to modify the cashier, although it should modify the customer. In order to correct this, we must place the modifying phrase directly next to what it modifies • Correct: Upon leaving the register, the customer received a receipt from the cashier Example # 2 • Incorrect: Kendra is happy, like all her friends, to be on vacation • Correct: Kendra, like all her friends, is happy to be on vacation • Incorrect: Unskilled in complex math, Bill’s score on the entrance exam was poor • Correct: Unskilled in complex math, Bill did not score well on the entrance exam
  • 21. MODIFIERS – 2 RELATIVE PRONOUNS - which, that, where, who, whose, whom Relative pronouns introduce relative clauses, which are a type of dependent clause. Relative clauses modify a word, phrase, or idea in the main clause Who introduces phrases that modify a person or a group of people (if the antecedent is in the object, then whom is to be used) Which introduces phrases that modify things That can be used to modify either people or things Where can be used to modify to places In which = places/ situations/time period/ phenomenon Example Incorrect: The Yankees, never liking to lose, practice everyday Correct: The Yankees, who never like to lose, practice everyday Correct: One of the books, which were on the table is mine • That is preferred to which • That is typically used to refer to singular nouns, and those/these is used to refer to plural nouns • ,which ,that ,who ,whom ,it, whose ,it - refers to the immediately preceding noun, and not to the action of the entire preceding clause Example • The police found the murder weapon, which made the prosecutor’s job much easier • The above sentence literally means that the murder weapon itself rather than finding the murder weapon made the prosecutor’s job easier • Better: The police found the murder weapon making the prosecutor’s job much easier
  • 22. MODIFIERS – 3 ESSENTIAL VS. NON-ESSENTIAL MODIFIERS “Which” is used to introduce non-essential modifiers – can be removed from the sentence without the sentence losing any essential meaning “That” is used to introduce essential modifiers – essential to the meaning of the sentence Commas are used to separate non-essential modifies from the noun that is modified • Non-essential: To find my house, walk down the left side of the street until you reach the third house, which is red • Essential: To find my house, walk down the left side of the street until you reach the third house that is red • Non-essential: This is my uncle John, who lives in Toronto • Essential: This is my uncle John who lives in Toronto • Only guests who are accompanied by tenants may use the pool • Only guests, who are accompanied by tenants, may use the pool • The first sentence identifies a specific group of guests who may use the pool: those accompanied by tenants (as opposed to those unaccompanied by tenants). By contrast, the second sentence indicates (somewhat illogically) that only guests (as opposed to tenants) may use the pool, and that they just happen to be accompanied by tenants. The second sentence contains a non-essential clause where an essential clause is required AND/BUT/ALTHOUGH When there is an and/but/although in a sentence, then the pronoun refers to the first noun or the main subject • Although Mary is younger than Susan, she is more mature
  • 23. PARALLELISM – 1 PARALLEL STRUCTURES Nouns Trevor collects stamps, coins and cards Adjectives The wait staff was prompt, friendly and competent Modified nouns A positive attitude can lead to both practical success and spiritual fulfillment Verbs We worked all day, ate all evening, and slept all night Verb infinitives I decided to swim across the river than sail around the world (the second to is optional) Participial phrases The rain continued to fall, providing water for thirsty plants but flooding the streets as well Adverbs I’ve noticed that you often howl angrily after you cower fearfully Adverbial phrases I’ve noticed that you often howl in anger after you cower in fear If one item includes a pronoun, it is often appropriate to include the same pronoun in all items • Incorrect: I prefer to hire employees who work hard to those that don’t • Correct: I prefer to hire employees who work hard to those who don’t Only structures that are logically parallel must be structurally parallel • Correct: Ken traveled around the world, visiting historic sites, eating native foods, and learning about new cultures • Incorrect: Ken traveled around the world, visited historic sites, ate native foods, and learned about new cultures • The incorrect version gives all activities equal emphasis, instead of making the last three activities subordinate to the main activity of traveling around the world CAUSE – EFFECT PARALLELISM Whenever there is a cause – effect in a sentence, the effect must begin with an –ing form of the verb after the comma • The stock markets are strong, giving us better forecasts • The rains were heavy, leading to some crop damage • The principal declared the results, making everyone cheer
  • 24. PARALLELISM – 2 Verbs of Being – Be aware Instead of expressing what the subject does, these verbs express what a subject is, or the condition a subject is in Most common form of the verb being is “to be” Forms of “to be” are – is, am, are, was, were, been, being Other common words of being – appear, become, feel, grow, look, remain, seem, smell, sound, stay, taste, turn • Incorrect: The flower bouquet WAS the husband’s giving of love to his wife • Correct: The flower bouquet WAS the husband’s loving gift to his wife • The two sides of the being word “was” are flower bouquet and husband’s giving of love. In order to achieve parallelism, rewrite the sentence replacing giving with the noun gift Idioms with built-in parallel structure – the word immediately after the first part of the pair will be the same as the word immediately following the second part of the pair (Example: Not only in… but also in.. ) • More X than Y • The more X the greater Y • No less was X than was Y • As X to Y • Not only X but also Y • Not X but rather Y • X instead of Y • The same to X as to Y • Range from X to Y • Both X and Y • Mistake X for Y • Prefer X to Y • X regarded as Y • To think of X as Y • Believe X to be Y • Between X and Y • Either X or Y • As X as Y • Just as X so Y (Example: Just as gills are to fish, so lungs are to humans) • Not so much X as Y (Example: Not so much to show Jane up as to make her appear foolish, Sarah pointed out Jane’s error to their supervisor) • More/greater/fewer less – than • Same X as Y • Just as X, so to Y (Example: Just as stealing is frowned upon, so too is cheating) Articles are parallel – a, an, the
  • 25. COMPARISON – 1 Category Example Comparison signals like, unlike, likening, more than, greater than, less than, shorter than, different from, as, as (adjective) as, as many as, as few as, as much as, as little as, as high as, as short as “Like” vs. “As” Like should be used to compare people or things (any nouns)  Incorrect: Bella and June, AS their mother, are extremely smart  Correct: Bella and June, LIKE their mother, are extremely smart As/as if should be used in a comparison involving clauses (any phrase that includes a verb)  Incorrect: Just LIKE swimming is good exercise, skiing is a great way to burn calories  Correct: Just AS swimming is good exercise, skiing is a great way to burn calories Parallelism – must compare similar things and be structurally parallel  Incorrect: Frank’s build, like his brother, is broad (here, frank’s build is being compared to his brother. This is not logical as it does not compare two similar things)  Correct: Frank’s build, like that of his brother, is broad  Incorrect: The host paid more attention to his business clients than others  Correct: The host paid more attention to his business clients than to others  Incorrect: Mary is taller than any girl in class (this implies that Mary is even taller than herself)  Correct: Mary is taller than any other girl in class  Incorrect: I enjoy flying planes more than I like to drive a car  Correct: I enjoy flying planes more than I like driving cars
  • 26. COMPARISON – 2 Category Example Unclear comparisons  Incorrect: Byron admired Dryden more than Wordsworth  Correct: Byron admired Dryden more than he did Wordsworth  Correct: Byron admired Dryden more than Wordsworth did Incomplete comparisons Incomplete comparisons are normally corrected by inserting a phrase like those of, those in, those at, that of, that in, and that at  Incorrect: The peaches here are riper than other fruit stand (peaches are being compared with an unlike thing – fruit stands)  Correct: The peaches here are riper than those at other fruit stand Like/Unlike Compare similar things. The word after like/unlike must match the word after the comma  Like Mary, John has a black car  Unlike Mary, Susan is tall Compared to vs. compared with Compared to (comparison between unlike things) vs. compared with (comparison between like things)  He compared her to a summer day  The police compared the forged signature with the original Use comparative form when comparing two things; the superlative form when comparing 3 or more things
  • 27. IDIOMS – 1 Correct Usage Incorrect Usage Awareness of Awareness about To regard X as Y To regard X as to regard Y So X as to Y (emphasis is on X) Example: So tall as to touch the ceiling Enough X to Y (emphasis is on Y) Example: Tall enough to touch the ceiling Considered/consider Example: “Scientists consider control factors an integral element…” Considered to be Example: “Scientists consider control factors to be an integral element…” Declare XY Example: Declared the Roman aqueduct of Segovia a Heritage of Humanity in 1985 Declare X to be Y Example: Declared the Roman aqueduct of Segovia to be a Heritage of Humanity in 1985 In the vicinity of Around the vicinity of Prices can only be higher or lower This price is more expensive than that one Agree with another person Agree to/upon something In contrast to In contrast with Regarded as Regarded to be When “rates” refer to the price charged, it should be followed by “for” Example: “Rates for telephone service have increased in recent years.” Thinking words such as “theory, belief ” or “believe” are often followed by “that” Example: Lucy’s belief that the Holocaust did not occur is misguided Persuade X to do Y Persuade X from doing Y
  • 28. IDIOMS – 2 Correct Usage Incorrect Usage Apprenticeship as Apprenticeship of being Dispute over Dispute about / dispute concerning Attempted to Attempted transparent enough to be transparent as to be transparent enough so as to be Estimated to be Estimated at Forbid X to do Y Forbid X from doing Y Prefer X to Y Prefer X over Y Consider XY Many financial analysts consider an upward trend in a firm’s current ratio a sign of improving liquidity Consider X as Y Many financial analysts consider an upward trend in a firm’s current ratio as a sign of improving liquidity Remembered for Remembered because of
  • 29. ODDS AND ENDS – 1 RULE # 1 – Countable vs. uncountable • Countable items include dollars, hats, people and buildings • Uncountable items include money, water, wreckage and patience - Always takes a singular verb in a sentence Countable modifiers Uncountable modifiers Examples Many Much As many as As much as Few/fewer Little/less • Incorrect: The company fired no less than fifty employees • Correct: The company fired no fewer than fifty employees Number of Amount of RULE # 2 – Among vs. Between Relating to two things Relating to three or more things Between X and Y Among X, Y and Z X is better than Y X is the best (among x, y, and z) X has more than Y X has the most (among x, y and z) X has less than Y X has the least (among x, y and z)
  • 30. ODDS AND ENDS – 2 Increase and decrease vs. greater and less Increase and decrease represent the change of ONE thing over time Greater or less signals a comparison between TWO things Greater than is correct only when it is used to describe numbers alone (greater than 10). If a sentence measures a percent rather than solely a number, the correct option is more than  The price of silver increased by 10 dollars  The price of silver is greater than the price of copper Connecting words Be aware of sentences that have no logical connectors between 2 independent clauses. Common connecting words are: and, or, nor, but, yet, although, when, because, for, since, before, after, if, & unless  Incorrect: I need to relax, I have so much to do (run-on sentence)  Correct: I need to relax, but I have so much to do Connecting punctuation The semicolon ( ; ) is used to connect two closely related statements. BOTH statements need to stand together as independent sentences The colon ( : ) is used to equate two parts of a sentence, where the second part is dependent on the first part. It is often used to equate a list with its components. One should be able to insert the word “namely” after the colon. Only the statement that precedes the colon must be able to stand alone  Incorrect: Andrew and Lisa are inseparable; doing everything together (second part of the sentence is incapable of standing on its own)  Correct: Andrew and Lisa are inseparable; they do everything together  Incorrect: I love listening to: classical rock, rock and pop music  Correct: I love many kinds of music: [namely] classical rock, rock and pop music If/Whether Whether is correct when you're discussing two options (whether to get chocolate or strawberry ice cream). On the GMAT, whether will (almost) always beat if Only/once Words with only/once always refer to the item coming immediately after them  Incorrect: Existed once  Correct: Once existed
  • 31. ODDS AND ENDS – 3 Because vs. In that When ‘because’ and ‘in that’ are in a sentence, more often than not, ‘in that’ would be correct. ‘In that’ qualifies the previous sentence, while ‘because’ is just used to show a simple causal relationship Teratomas are unusual forms of cancer because they are composed of tissues such as tooth and bone not normally found in the organ in which the tumor appears. A. because they are composed of tissues such as tooth and bone B. because they are composed of tissues like tooth and bone that are C. because they are composed of tissues, like tooth and bone, tissues D. in that their composition , tissues such as tooth and bone, is E. in that they are composed of tissues such as tooth and bone, tissue Can vs. Could If you are just assuming something, ‘could’ should be used Could is used for: - Possibility (John could be the one who stole the money) - Condition (If I had more time, I could travel around the world) - Suggestion (You could spend your vacation here) - Polite request (Could I have something to drink?) Artificial intelligence emerged during the late 1950's as an academic discipline based on the assumption that computers are able to be programmed to think like people A. are able to be programmed to think like people B. were able to be programmed to think as people C. can be programmed to think as people can D. could be programmed to think like people E. are capable of being programmed to think like people do Usual vs. Is usual When something is compared to a subgroup to which it belongs, is usual should be used. When something is compared to itself, usual is fine  Incorrect: A Mercedes is more expensive than usual for a car  Correct: A Mercedes is more expensive than is usual for a car  He is nicer than usual The If we use ‘the’ we are saying that there are only 50 million Turks in the whole world; if we don’t use ‘the’ we’re saying that there are possibly more than 50 million Turks in the world • Although about 99% of the more than 50 million Turks are Muslims • Although about 99% of more than 50 million MISCELLANEOUS • “Because of” is used to modify an adverbial phrase (verb/verb phrase) • “Despite” requires a noun or a noun phrase • Just as can replace “in the same way that” • "instead of" is reserved for nouns and "rather than" for verbs
  • 32. THINGS TO AVOID • Noun forms (words ending in -ance, -ence, -ment, -ion,-ity) • Because of • Apostrophe • Passive voice WORDS/PHRASES THAT ARE ALMOST ALWAYS INCORRECT • Do it (use do so) - Incorrect: He said that I ate his cookies, but I didn’t do it • The numbers of - Incorrect: The politicians were amazed at the numbers of anti-war protestors • Whether or not (just use whether) - Incorrect: He couldn’t decide whether or not to apply to Harvard • Due to (should only be used if it can be replaced by ‘caused by’; used to modify a noun) - Incorrect: the game was postponed due to rain - Correct: His failure was due to his laziness - Incorrect: He failed due to his laziness • So as to (use such that) • Just like • On account of • Occurring (to be used only for natural calamity) • Such like • Owing to • Lesser • Hopefully • Seeming • Not any • Being/ having
  • 34. IMPORTANT NOTES • When reading Logical Reasoning stimuli, you should seek to make several key determinations: – Does the stimulus contain an argument or is it only a set of factual statements? – If the stimulus contains an argument: • What is the conclusion? • Is the argument strong or weak? To determine the strength of the argument, consider the relationship between the premises and the conclusion—do the premises strongly suggest that the conclusion would be true? Does the conclusion feel like an inevitable result of the premises? Or does the conclusion seem to go beyond the scope of the information in the premises? DO THE FACTS SUPPORT THE CONCLUSION? – Read the fine print! • Quantity and Probability Indicators • Scope – Carefully read and identify the question stem. Do not assume that certain words are automatically associated with certain question types – One of the most effective techniques for quickly finding correct answer choices and avoiding incorrect answer choices is pre- phrasing. Pre-phrasing an answer involves quickly speculating on what you expect the correct answer will be based on the information in the stimulus • INFER = MUST BE TRUE
  • 35. PREMISE ARGUMENT STIMULUS Indicators Definition A fact, proposition, or statement from which a conclusion is made Premises support and explain the conclusion Example: The Jacksonville area has just over one million residents. The Cincinnati area has almost two million residents. The New York area has almost twenty million residents because, since, for, for example, for the reason that, in that, given by, as indicated by, due to, owing to, this can be seen from, we know this by Additional premise indicators (another premise that supports the conclusion but is sometimes non- essential to the conclusion) – furthermore, moreover, besides, in addition, what’s more thus, therefore, hence, consequently, as a result, so, accordingly, clearly, must be that, shows that, conclude that, follows that, for this reason Premise + Conclusion Conclusion is a statement or judgment that follows from one or more reasons Conclusion = premise + assumption (implicit) Example: All professors are ethical. Mason is a professor. So Mason is ethical. The first two statements in this argument give the reasons (or “premises”) for accepting the third statement, which is the conclusion Imp: Premises and conclusions can be presented in any order; the order has no effect on the logical structure of the argument. Similarly, it is not necessary for the premise and conclusion to be split into multiple sentences Tip: The conclusion is generally the first or last sentence of a stimulus
  • 36. DETERMINING PREMISES AND CONCLUSIONS Example Premise Conclusion (may be added in the question stem ) “The rapid diminishment of the ecosystem of the Amazon threatens the entire planet. Consequently, we must take immediate steps to convince the Brazilian government that planned development projects need to be curtailed for the simple reason that these development projects will greatly accelerate the loss of currently protected land.” Premise 1: “The rapid diminishment of the ecosystem of the Amazon threatens the entire planet.” Premise 2: “for the simple reason that these development projects will greatly accelerate the loss of currently protected land.” We must take immediate steps to convince the Brazilian government that planned development projects need to be curtailed Every professor at Fillmore University teaches exactly one class per semester. Fillmore’s Professor Jackson, therefore, is teaching exactly one class this semester. Moreover, I heard Professor Jackson say she was teaching only a single class Every professor at Fillmore University teaches exactly one class per semester The first sentence is the main proof offered by the author for the conclusion. The third sentence begins with the additional premise indicator “moreover.” The premise in this sentence is non-essential to the argument, but provides additional proof for the conclusion and could be, if needed, used to help prove the conclusion separately (this would occur if an objection was raised to the first premise) Fillmore’s Professor Jackson, therefore, is teaching exactly one class this semester Humans cannot live on Venus because the surface temperature is too high The surface temperature is too high Humans cannot live on Venus Therefore, since higher debt has forced consumers to lower their savings, banks now have less money to loan Higher debt has forced consumers to lower their savings Banks now have less money to loan Television has a harmful effect on society. This can be seen from the poor school performance of children who watch significant amounts of television and from the fact that children who watch more than six hours of television a day tend to read less than non-television watching children Premise 1: This can be seen from the poor school performance of children who watch significant amounts of television Premise 2: and from fact that children who watch more than six hours of television a day tend to read less than non-television watching children Television has a harmful effect on society.
  • 37. TYPES OF CRITICAL REASONING QUESTIONS PROVE HELP HURT Must accept the stimulus – even if it contains an error in reasoning – and use it to prove that one of the answer choices must be true Any information in an answer choice that does not appear either directly in the stimulus or as a combination of the items in the stimulus will be incorrect Reasoning errors are often present in the stimulus; you may use the “new” information provided in the answer choices to strengthen/support the argument Reasoning errors are often present in the stimulus; you may use the “new” information provided in the answer choices to weaken the argument Must be true/ Inference Assumption Weaken Main point Strengthen/support Method of reasoning Paradox Flaw in reasoning Parallel reasoning
  • 38. A. MUST BE TRUE/ INFERENCE Paraphrased answers - answers that restate a portion of the stimulus in different terms. Because the language is not exactly the same as in the stimulus, paraphrased answers can be easy to miss CORRECT ANSWERS Combination answers - Answers that result from combining two or more statements in the stimulus Things to keep in mind - The stimulus comprises a set of facts ONLY, and the objective is to determine the conclusion based on the facts presented. NO ADDITIONAL ASSUMPTIONS MUST BE MADE - Be careful of indicator and modifier words such as entirely, some, almost, all, could lead to, many , not, but . They define the scope (narrow or broad) and the tone of the argument. INCORRECT ANSWERS Answers that are possible but not certain Because the criteria in the question stem requires you to find an answer choice that Must Be True, answers that only could be true or are even likely to be true are incorrect. These answers are attractive because there is nothing demonstrably wrong with them (for example, they do not contain statements that are counter to the stimulus) Exaggerated answers These answers take information from the stimulus and then stretch that information to make a broader statement that is not supported by the stimulus For example, if the stimulus stated that “Some software vendors recently implemented more rigorous licensing procedures”, an incorrect answer would exaggerate one or more of the elements: “Most software vendors recently implemented more rigorous licensing procedures.” Answers that present so-called “new information” For example, if the stimulus discusses the economic policies of Japan, be careful with an answer that mentions U.S. economic policy. Look closely at the stimulus—does the information about Japanese economic policy apply to the U.S.? Do not eliminate if the answer falls within the sphere of a term or concept in the stimulus, or if it is a consequence of combining stimulus elements Reverse answers The stimulus might state, “Many people have some type of security system in their home.” An incorrect answer then reverses the elements: “Some people have many types of security systems in their home.” Shell Game Occurs when an idea or concept is raised in the stimulus and then a very similar idea appears in the answer choice, but the idea is changed just enough to be incorrect but still attractive Opposite answer Opposite of the stated facts of the stimulus. Example, stimulus shows that X caused Y, but answer choice states that Y caused X When a stimulus contains only the opinions of others, then in a Must Be True question you can eliminate any answer choice that makes a flat assertion without reference to those opinions
  • 39. MUST BE TRUE – CONTINUED Flavonoids are a common component of almost all plants, but a specific variety of flavonoid in apples has been found to be an antioxidant. Antioxidants are known to be a factor in the prevention of heart disease Which one of the following can be properly inferred from the passage? A. A diet composed largely of fruits and vegetables will help to prevent heart disease B. Flavonoids are essential to preventing heart disease C. Eating at least one apple each day will prevent heart disease D. At least one type of flavonoid helps to prevent heart disease E. A diet deficient in antioxidants is a common cause of heart disease 1st Statement: Flavonoids are a common component of almost all plants 2nd Statement: A specific variety of flavonoid in apples has been found to be an antioxidant 3rd Statement: Antioxidants are known to be a factor in the prevention of heart disease A The answer choice could be true, but it is too broad to be supported by the facts: nowhere are we told that a diet of fruits and vegetables will help prevent heart disease. Perhaps apples are the only fruit with the antioxidant flavonoid and there is nothing beneficial about other fruits and vegetables. And, eating a diet of fruits and vegetables is no guarantee that the diet includes apples. Regardless, this answer choice can be especially attractive because it plays on the general perception that fruits and vegetables are good for you B Nothing in the stimulus supports the rather strong statement that flavonoids are essential to preventing heart disease C This answer is incorrect because the language is too strong: the stimulus only stated that apples contain an element that was a factor in preventing heart disease, not that they definitely will prevent heart disease D This answer is the closest to our pre-phrase, and this is the correct answer. Notice how the language of this answer choice—“helps to prevent”—matches the stimulus language— “factor in the prevention.” E This answer choice also could be true, but it cannot be correct because the stimulus makes no mention of the causes of heart disease. Just because an antioxidant can help prevent heart disease does not mean that a lack of antioxidants causes heart disease
  • 40. B. MAIN POINT Example: Journalist: A free marketplace of ideas ensures that all ideas get a fair hearing. Even ideas tainted with prejudice and malice can prompt beneficial outcomes. In most countries, however, the government is responsible for over half the information released to the public through all media. For this reason, the power of governments over information needs to be curtailed. Everyone grants that governments should not suppress free expression, yet governments continue to construct near monopolies on the publication and dissemination of enormous amounts of information Which one of the following most accurately expresses the conclusion of the journalist’s argument? (A) The freedom of the marketplace of ideas is in jeopardy. (B) Preserving a free marketplace of ideas is important (C) The control that governments have over information needs to be reduced (D) Ideas that have malicious content or stem from questionable sources can be valuable (E) Governments have near monopolies on the dissemination of many kinds of information (A) The author would agree with this statement but this is not the Main Point of the argument; rather, it is closer to a premise that might support the conclusion (B) The author believes that the freedom of the marketplace of ideas is at risk, and in stating that we should curtail the government’s power over information, the author assumes that preserving a free marketplace of ideas is important. Thus this answer choice would be better described as an unstated premise that supports the conclusion (C) This is the correct answer. Remember, any answer that is a paraphrase of the conclusion of the argument will be the correct answer to a Main Point question (D) The stimulus specifically notes that malicious or prejudicial ideas can “prompt beneficial outcomes.” The outcome of an idea is different than stating the ideas themselves “can be valuable.” (E) The stimulus states that “governments continue to construct near monopolies on the publication and dissemination of enormous amounts of information.” This phrasing is not the same as answer choice (E), which asserts that the government already has a monopoly on the dissemination of many kinds of information Primary objective Question identifiers Incorrect answers Answers that are true but do not encapsulate the author’s point Answers that repeat premises of the argument “Which one of the following most accurately expresses the main conclusion of the argument?” “Which one of the following most accurately restates the main point of the argument” To identify the CONCLUSION based ONLY on facts/premises explicitly listed in the stimulus
  • 41. C. WEAKEN Question Identifiers “Which of the following most seriously undermines the argument?” “Which of the following, if true, calls into question the validity of the argument?” Trigger words: Weaken, attack, undermines, contradicts, evidence against, challenge, damage, counter, refute etc. Primary Objective There is always a flaw in the reasoning of the stimulus (gross generalization, conclusions drawn from incomplete information, improper comparison etc.) To solve these questions, you first need to identify the premise and the conclusion. In this question type, we assume an answer choice presented to be true – even if it introduces new information (obviously, the information has to be relevant to the stimulus) Incorrect answers • Opposite answers – answers that strengthen rather than weaken the argument • Shell game answers – similar idea to that of the stimulus, but not entirely true • Improper comparison – comparing two or more items that are essentially different • Out of scope answers • Wrong tone answers • Reversal of causality/incorrect causality Cause & Effect Reasoning - Causality occurs when one event is said to make another occur. Just because one event occurs before the other, or that two events occur simultaneously, does not mean that there is a cause-effect relationship - Words that indicate cause-effect relationship: cause by, because of, responsible for, reason for, leads to, induced by, promoted by, determined by, produced by, product of, played a role in, was a factor in, is an effect of - The question assumes that the only cause is the one stated in the conclusion and that there are no other causes that can create that particular effect - In weaken questions, the correct answer choice with do any of the following: • Find an alternate cause for the stated effect. Because the author believes there is only one cause, identifying another cause weakens the conclusion • Show that even when the cause occurs, the effect does not occur • Show that although the effect occurs, the cause did not occur • Show that the stated relationship is reversed (the claimed effect is actually the cause of the claimed cause) • Show that a statistical problem exists with the data used to make the casual statement Example The consumption of ice cream has been found to correlate with the murder rate. Therefore, consuming ice cream must cause one to be more likely to commit murder However, this argument fails to take into account that a third element, hot weather, could be the cause of the increase in murder rate and in the consumption of ice cream
  • 42. WEAKEN – CONTINUED Robot satellites relay important communications and identify weather patterns. Because the satellites can be repaired only in orbit, astronauts are needed to repair them. Without repairs, the satellites would eventually malfunction. Therefore, space flights carrying astronauts must continue. Which of the following, if true, would most seriously weaken the argument above? A. Satellites falling from orbit because of malfunctions burn up in the atmosphere B. Although satellites are indispensable in the identification of weather patterns, weather forecasters also make some use of computer projections to identify weather patterns C. The government, responding to public pressure, has decided to cut the budget for space flights and put more money into social welfare programs D. Repair of satellites requires heavy equipment, which adds to the amount of fuel needed to lift a spaceship carrying astronauts into orbit E. Technical obsolescence of robot satellites makes repairing them more costly and less practical than sending new, improved satellites into orbit A Irrelevant B Doesn’t provide a reason for not sending astronauts to space. Out of scope C Though this seems like a right answer, think about it. This is lots of irrelevant information, and doesn’t answer our question directly D Fuel? This is a shell game fallacy. Clearly shows a statement that could be true and is very attractive to the test taker, but irrelevant to what’s asked E This makes sense. If repair cost > new satellite cost, why send astronauts to space to repair them? Just send new satellites Violent crime in this town is a becoming a serious problem. Compared to last year, local law enforcement agencies have responded to 17 per cent more calls involving violent crimes, showing that the average citizen of this town is more likely than ever to become a victim of a violent crime. Which one of the following, if true, most seriously weakens the argument? A. The town’s overall crime rate appears to have risen slightly this year compared to the same period last year B. In general, persons under the age of 65 are less likely to be victims of violent crimes than persons over the age of 65 C. As a result of the town’s community outreach programs, more people than ever are willing to report violent crimes to the proper authorities D. In response to worries about violent crime, the town has recently opened a community centre providing supervised activities for teenagers E. Community officials have shown that a relatively small number of repeat offenders commit the majority of violent crimes in the town A This is an opposite answer that strengthens the argument B Because the argument is about “the average citizen of this town,” information about victims of a certain age is irrelevant C This is the correct answer. By showing that people are more willing to report crimes (and thus call them in for response), an alternate cause for the rise in the number of calls is given D This answer only addresses an effect of the concern over crime, and does not address the causal relationship that underlies the argument E This answer does not address a possible rise in crime or the reasons for the rise in responses to calls involving violent crime
  • 43. D. STRENGTHEN Question Identifiers Stem would indicate some kind of “support” relationship – strengthen, justify, help, support etc. ** “The stimulus supports which of the following” would be an inference/main point question as opposed to the “strengthen” question which would point to “the answer choice supporting the stimulus” – subtle but important difference Primary Objective Identify the conclusion—this is what you are trying to strengthen! Find the missing link between a premise and the conclusion (typically an assumption) Important to keep in mind Don’t try to disprove a conclusion Incorrect answers • Opposite answers – answers that weaken rather than strengthen the argument • Out of Scope • Shell Game • Wrong tone answers • Answers that merely state the premise Correct answers • Connect evidence with conclusion better • Make the conclusion stronger • Arguments that contain analogies or use surveys rely upon the validity of those analogies and surveys. Answer choices that strengthen the analogy or survey, or establish their soundness, are usually correct • Strengthen the evidence with new information (perhaps an assumption is needed to make the argument work
  • 44. STRENGTHEN – CONTINUED Dr. Larson: Sleep deprivation is the cause of many social ills, ranging from irritability to potentially dangerous instances of impaired decision making. Most people suffer from sleep deprivation to some degree. Therefore we should restructure the work day to allow people flexibility in scheduling their work hours Which of the following, if true, would most strengthen the medical doctor’s argument about sleep deprivation? The conclusion of the argument is straightforward. Allow flexibility in scheduling work hours. The basic underlying assumption in the conclusion basically tells you that the author is assuming that greater flexibility would promote better sleeping times. Now we can have this in our mind as we look for answers. A. The primary cause of sleep deprivation is overwork B. Employees would get more sleep if they had greater latitude in scheduling their work hours C. Individuals vary widely in the amount of sleep they require D. More people would suffer from sleep deprivation today than did in the past if the average number of hours worked per week had not decreased E. The extent of one’s sleep deprivation is proportional to the length of the workday A Perfect shell game choice. So this choice is telling you that overwork causes sleep deprivation. We are not talking about the quantity of work here, but instead the flexibility of scheduling. Irrelevant B Correct C Completely irrelevant. We are not talking about how much sleep an individual requires at all. It presents new information but without any connection D This talks about a decrease in work hours and in a very convoluted structure. Are we talking about the number of hours worked? No. We are talking only about the flexibility of scheduling E Once again, length is immaterial
  • 45. E. ASSUMPTIONS Question Identifiers “The conclusion depends on which of the following?” “The author assumes that?” “Based on” “Assumption Made” “Cannot be true unless?” Primary Objective An assumption is an unstated premise and is NECESSARY to arrive at a conclusion Find the missing link between a premise and the conclusion (typically an assumption) Incorrect answers • Contain extra information - For example, let’s say that an argument requires the assumption that “all dogs are intelligent”. The correct answer could be that statement or even a subset statement such as “all black dogs are intelligent” or “all large dogs are intelligent”. However, in the case of the statement “all dogs and cats are intelligent”, the additional information about cats is not part of the authors assumption, and would make the answer choice incorrect • Answers that claim an idea was the most important consideration for the author • Answer choice that repeats the premise Difference between Strengthen & Assumption Questions • Strengthen questions ask you to support the argument in any way possible. This type of answer has great range, as the additional support provided by the answer choice could be relatively minor or major. Speaking in numerical terms, any answer choice that strengthens the argument, whether by 1% or by 100%, is correct • Assumption questions ask you to identify a statement that the argument assumes or supposes. An assumption is simply an unstated premise— what must be true in order for the argument to be true. An assumption can therefore be defined as what is necessary for the argument to be true • Example: An argument concludes that a teenager is an outstanding golfer. In an Assumption question, the correct answer could be: “The teenager always hits the ball” or “The teenager never swings and misses the ball.” Either statement is an assumption of the argument; otherwise how could the teenager be an outstanding golfer? In a Strengthen question, the correct answer could be: “The teenager won a local club tournament.” This answer choice supports the idea that the teenager is an outstanding golfer, but does not undeniably prove the teenager to be outstanding (what if the tournament was composed primarily of pre-teen players?) nor is the answer an assumption of the conclusion
  • 46. CORRECT ASSUMPTION ANSWER CHOICE SUPPORTER • Links together new/ rogue elements in the stimulus • Conclusion often contains a piece of information not previously included in the stimulus • Fills logical gaps in the argument Example: All male citizens of Athens had the right to vote. Therefore, Socrates had the right to vote in Athens Assumption: Socrates was male DEFENDER Eliminate ideas or assertions that would undermine the conclusion. In this sense, they “defend” the argument by showing that a possible source of attack has been eliminated Example: People who read a lot are more intelligent than other people. Thus, reading must cause a person to be intelligent Assumption: All other alternative explanations (such as regular exercise, high-protein diet, genetics etc.) are assumed not to exist Watch for answers starting with the phrase “at least one” or “at least some.” For some reason, when an Assumption answer choice starts with either of the above constructions the chances are unusually high that the answer will be correct ASSUMPTION NEGATION TECHNIQUE Basically involves converting the assumption question into a weaken question. The technique can only be applied to assumption questions, so you need to be careful, but once you’ve eliminated other possible answers, follow this technique to check the remaining options: • Negate the answer choice – This basically asks you to assume the opposite of whatever is given in the answer choice ( I went to the beach yesterday will become I did not go to the beach everyday last week; sweet will become not sweet; all the missions succeeded will become not all of the missions succeeded) ** All = 100; Not All = 0-99; Some = 1-100; None = 0 • Does the negated answer choice make the conclusion collapse? – If the answer is yes, then the answer choice is the right one. Any negated answer choice that attacks the conclusion or questions its validity is one that is the right answer.
  • 47. F. RESOLVE THE PARADOX Question Identifiers “Which of the following, if true, most helps resolve the apparent paradox?” Resolve/ reconcile/ Explain the following – Discrepancy/ contradiction/ conflict/ puzzle Primary Objective Two contradictory facts are presented as they are. There is no inference/conclusion. You must resolve the paradox (i.e., allow both sides to be factually correct) Incorrect answers • Answer explains only one side of the paradox • Does not conform to the specifics of the stimulus • If the stimulus contains a paradox where two items are similar, then an answer choice that explains a difference between the two cannot be correct and vice versa. The answer choice must resolve the paradox, not strengthen or weaken it Correct Answers Correct answers will actively resolve the paradox, that is, it will allow both sides to be factually correct and it will either explain how the situation came into being or add a piece of information that shows how the two ideas or occurrences can co-exist Park Ranger: When snowfall levels are below average during winter months, scattered patches on the forest floor often remain exposed and accessible to scavenging wildlife. Because squirrels are able to collect nuts only in the snow-free areas of the forest, the squirrel population tends to increase when there is a below average snowfall. However after last year’s unprecedented snow-free winter season, the squirrel population in this region was determined to be a 20 year low Which of the following, if true, helps explain the paradox above? A. When snowfall is above average, squirrel populations tend to diminish as squirrels are unable to forage for food in snow-covered areas B. The squirrels’ spring breeding season does not begin until all of the snow in the forest has melted. C. The red-tailed hawk, the squirrel’s most common predator, does not migrate south out of the forest until the first snowfall of the winter season. D. Forest squirrels rarely feed on berries or fruits and prefer nuts for their high calorific content E. The current system of estimating squirrel population size is thought to be extremely accurate in its projections A Irrelevant. The first phrase itself tells us this is useless since we are looking to explain an event that happens during a snow-free winter B This doesn’t really explain anything either. If at all anything, the population should have gone up. C This seems good. If the predator doesn’t move until the snowfall, and there was no snowfall, then the squirrel population would have gone down D Irrelevant E Irrelevant
  • 48. G. BOLD FACED QUESTIONS Question Identifiers “The two bold faced sentences play which of the following roles?” Primary Objective These questions are heavily dependent on interpreting the premise and identifying the conclusion of the stimulus correctly and hence can be considered similar to an inference/must be true question type. To correctly answer the question it’s important to identify the pattern of reasoning in the stimulus correctly. Thus, going back to the inference lesson, for this question type, we need to isolate the conclusion, understand the premise and identify any logical errors in the drawing of the conclusion Incorrect answers • Answer explains only one side of the bold faced questions • Does not conform to the specifics of the stimulus Historian: Newton developed mathematical concepts and techniques that are fundamental to modern calculus. Leibniz developed closely analogous concepts and techniques. It has traditionally been thought that these discoveries were independent. Researchers have however, recently discovered notes of Leibniz’ that discuss one of Newton’s books on mathematics. Several scholars have argued that since the book includes a presentation of Newton’s calculus concepts and techniques and since the notes were written before Leibniz’ own development of calculus concepts and techniques, it is virtually certain that the traditional view is false. A more cautious conclusion than this is called for, however. Leibniz’ notes are limited to early sections of Newton’s book; sections that precede the ones in which Newton’s calculus concepts and techniques are presented. In the historian’s reasoning, the two boldfaced portions play which of the following roles? This is a question where it’s absolutely necessary to identify what the author’s stand on the argument is, and hence identifying the conclusion becomes vital. The traditional view is that Leibniz and Newton developed these simultaneously. But people believe this is false because they found Leibniz mentioning that he’d studied Newton’s book. Look at what the historian says in response: A more cautious conclusion. So this means he doesn’t believe that Leibniz had written the works after Newton and hence justifies that by giving an alternative explanation. So to deconstruct this question we have two statements: Researcher’s conclusion: Traditional view is false. Historian’s conclusion: Researcher’s conclusion is false. A The first provides evidence in support of the overall position that the historian defends; the second is evidence that has been used to support an opposing position. The historian doesn’t defend the position, he opposes it. So incorrect B The first provides evidence in support of the overall position that the historian defends; the second is that position Once again, the historian doesn’t defend this position, hence incorrect. C The first provides evidence in support of an intermediate conclusion that is drawn to provide support for the overall position that the historian defends; the second provides evidence against that intermediate conclusion This is the kind of complicated wording that GMAT likes to trick us with. There is no intermediate conclusion, and the historian doesn’t support the overall conclusion D The first is evidence that has been used to support a conclusion that the historian criticizes; the second is evidence offered in support of the historian’s own position This is true. The historian opposes the first and gives an alternative explanation. E The first is evidence that has been used to support a conclusion that the historian criticizes; the second is further information that substantiates that evidence The second part of this is wrong. The first might be evidence that supports something the historian criticizes, however the second part is his alternative explanation and is not in support of the first. Hence incorrect
  • 49. H. METHOD OF REASONING Question Identifiers • “The argument proceeds by” • “The argument derives its conclusion by” • “Which one of the following is an argumentative strategy employed in the argument?” • “The argument employs which one of the following reasoning techniques?” Primary Objective Focus on the form of the argument instead of the concrete facts Incorrect answers • Describe an event that did not occur in the stimulus/ “New” Element Answers • Half right/half wrong – start by describing something that occurred in the argument and end by describing something that didn’t • Exaggerated/opposite/reversed answers Garbage in this neighbourhood probably will not be collected until Thursday this week. Garbage is usually collected here on Wednesdays, and the garbage collectors in this city are extremely reliable. However, Monday was a public holiday, and after a public holiday that falls on a Monday, garbage throughout the city is supposed to be collected one day later than usual. The argument proceeds by: (A) treating several pieces of irrelevant evidence as though they provide support for the conclusion (B) indirectly establishing that one thing is likely to occur by directly ruling out all of the alternative possibilities (C) providing information that allows application of a general rule to a specific case (D) generalizing about all actions of a certain kind on the basis of a description of one such action (E) treating something that is probable as though it were inevitable As usual, we begin by analyzing the structure of the problem: Premise: Garbage is usually collected here on Wednesdays, and the garbage collectors in this city are extremely reliable Premise: Monday was a public holiday Premise: After a public holiday that falls on a Monday, garbage throughout the city is supposed to be collected one day later than usual Conclusion: Garbage in this neighbourhood probably will not be collected until Thursday this week The argument is sound and the conclusion seems reasonable. The language in the conclusion is not absolute (“probably”), and this is justified since the language used in the argument—“usually” and “supposed to be”—is also probabilistic. Knowing that the argument is valid, the question you must ask yourself is, “How would I describe the structure of this argument?” A. This answer forces you to make an assessment of the premises (the “evidence”) as they relate to the conclusion. Are the premises irrelevant to the conclusion? Clearly not. Therefore, this answer is incorrect B. This is a half-right, half-wrong answer. The argument does establish “that one thing is likely to occur.” But, is this established by ruling out all of the alternative possibilities? No, to do that would mean presenting arguments against the garbage being collected on Friday, Saturday, Sunday, etc. Since this section of the answer choice does not occur, this answer is incorrect. Also, because the argument does not rule out all the alternatives, the conclusion is not established “indirectly.” C. This is the correct answer. Consider each piece of the argument: “providing information”—a variety of information about the garbage situation is provided. “application of a general rule”—the general rule is that “After a public holiday that falls on a Monday, garbage throughout the city is supposed to be collected one day later than usual.” “to a specific case”—the specific case is the pickup of garbage this week in this neighbourhood. Given that all elements occurred and the answer presents an accurate description of the way the author made his or her argument, this answer is correct. Now, take a moment and compare this answer to the pre- phrase you made after reading the stimulus. How similar are the two? Given that you may not be familiar with the language used by the test makers, the two may not be very similar. For example, note the use in this answer of “general rule” to describe the last sentence of the stimulus. The test makers could also have used a phrase like “basic principle” to achieve the same result. Your job is to match their language to what occurred in the stimulus D. This answer is an overgeneralization—a situation where one instance is used to make a broad based conclusion. This is a Reverse Answer since the stimulus actually uses a general principle and applies it to one instance E. This is an Exaggerated Answer. The conclusion states that “Garbage in this neighbourhood probably will not be collected until Thursday this week” and the use of “probably” is a clear and obvious indication that the author does not think the Thursday garbage pickup is inevitable
  • 50. I. FLAW IN REASONING Question Identifiers “The reasoning in the argument is most vulnerable to criticism on the ground that the argument “The reasoning above is flawed because it fails to recognize that” “The reasoning in the argument is fallacious because the argument” Primary Objective Flaw in the Reasoning questions are exactly the same as Method of Reasoning questions with the important exception that the question stem indicates that the reasoning in the stimulus is flawed. Because the question stem reveals that a flaw is present, you need not make a determination of the validity of the stimulus COMMON ERRORS OF REASONING Causal conclusion vs. causal claim made in the premise If the causal statement is the conclusion, then the reasoning is flawed Premise: In North America, people drink a lot of milk Premise: There is a high frequency of cancer in North America Conclusion: Therefore, drinking milk causes cancer In this case, the author takes two events that occur together and concludes that one causes the other If the causal statement is the premise, then the argument may be flawed, but not because of the causal statement Premise: Drinking milk causes cancer Premise: The residents of North America drink a lot of milk Conclusion: Therefore, in North America there is a high frequency of cancer among the residents **Uncertain Use of a Term or Concept “Some people claim that the values that this country was built on are now being ignored by modern-day corporations. But this is incorrect. Corporations are purely profit driven enterprises, beholden only to their shareholders, and as such they can only assess objects based on their value.” The term “value” is used in the example above in two different senses: first in a moral or ethical sense and then in a monetary sense. This shift in meaning undermines the author’s position - “depending on the ambiguous use of a key term” - “it confuses two different meanings of the word ‘solve’ - “relies on interpreting a key term in two different ways” - “equivocates with respect to a central concept” - “allows a key term to shift in meaning from one use to the next” - “fails to define the term”
  • 51. COMMON ERRORS OF REASONING – 2 Source Argument Attacks the person (or source) instead of the argument they advance. Because the GMAT is concerned solely with argument forms, a speaker can never validly attack the character or motives of a person; instead, a speaker must always attack the argument advanced by a person. Here is an example: “The anti-smoking views expressed by Senator Smith should be ignored. After all, Smith himself is a smoker!” A source argument can take different forms, including the following: 1. Focusing on the motives of the source 2. Focusing on the actions of the source (as in the above example) “makes an attack on the character of opponents” “it is directed against the proponent of a claim rather than against the claim itself” “he directs his criticism against the person making the argument rather than directing it against the argument itself” “it draws conclusions about the merit of a position and about the content of that position from evidence about the position’s source” “assuming that a claim is false on the grounds that the person defending it is of questionable character” Circular Reasoning In circular reasoning the author assumes as true what is supposed to be proved Example: “This essay is the best because it is better than all the others.” In this example the premise and the conclusion are identical in meaning. As we know, the conclusion should always follow from the premise. In the example above, the premise supports the conclusion, but the conclusion equally supports the premise, creating a “circular” situation where you can move from premise to conclusion, and then back again to the premise “it assumes what it seeks to establish” “argues circularly by assuming the conclusion is true in stating the premises” “presupposes the truth of what it sets out to prove” “the argument assumes what it is attempting to demonstrate” “it takes for granted the very claim that it sets out to establish” “it offers, in place of support for its conclusion, a mere restatement of that conclusion” Errors of Conditional Reasoning The authors can either mistake a necessary condition for a sufficient condition, or mistake a sufficient condition for a necessary condition “it treats something that is necessary for bringing about a state of affairs as something that is sufficient to bring about a state of affairs” “confuses a sufficient condition with a required condition”
  • 52. COMMON ERRORS OF REASONING – 3 Cause and effect - Assuming a causal relationship on the basis of the sequence of events - Assuming a causal relationship when only a correlation exists - Failure to consider an alternate cause for the effect, or an alternate cause for both the cause and the effect - Failure to consider that the events may be reversed - “mistakes a temporal relationship for a causal relationship” - “confusing the coincidence of two events with a causal relation between the two” - “assumes a causal relationship where only a correlation has been indicated” - “fails to exclude an alternative explanation for the observed effect” - “the author mistakes an effect for a cause” Straw man This error occurs when an author attempts to attack an opponent’s position by ignoring the actual statements made by the opposing speaker and instead distorts and refashions the argument, making it weaker in the process. Often this error is accompanied by the phrase “what you’re saying is” or “if I understand you correctly,” which are used to preface the refashioned and weakened argument - “refutes a distorted version of an opposing position” - “ incorrectly describing the student representative’s position, thereby making it easier to challenge” - “portrays opponents’ views as more extreme than they really are” - “distorts the proposal advocated by opponents” General Lack of Relevant Evidence for the Conclusion Failure to provide any information to support the conclusion/ irrelevant information Example: “Some critics claim that scientific progress has increased the polarization of society and alienated large segments of the population. But these critics are wrong because even a cursory glance at the past shows that society is always somewhat polarized and some groups are inevitably alienated.” - “The author cites irrelevant data.” - “draws a conclusion that is broader in scope than is warranted by the evidence advanced” - “It uses irrelevant facts to justify a claim about the quality of the disputed product” - “It fails to give any reason for the judgment it reaches.” - “It introduces information unrelated to its conclusion as evidence in support of that conclusion” Internal contradiction Example: “Everyone should join our country club. After all, it’s an exclusive group that links many of the influential members of the community.” - “bases a conclusion on claims that are inconsistent with each other” - “the author makes incompatible assumptions” - “introduce information that actually contradicts the conclusion” - “assumes something that it later denies, resulting in a contradiction” Appeal Fallacies Appeal to authority The flaw in this form of reasoning is that the authority may not have relevant knowledge or all the information regarding a situation, or there may a difference of opinion among experts as to what is true in the case. Example: use of a neurologist as an authority figure in an area of dentistry - “the judgement of experts is applied to a matter in which their expertise is irrelevant” - “the argument inappropriately appeals to the authority of the mayor” - “it relies on the judgment of experts in a matter to which their expertise is irrelevant” - “accepts a claim on mere authority, without requiring sufficient justification”
  • 53. COMMON ERRORS OF REASONING – 4 Appeal to numbers/percentage s/majority/popular opinion This error states that a position is true because the majority believes it to be true - “a claim is inferred to be false merely because a majority of people believe it to be false” - “the argument, instead of providing adequate reasons in support of its conclusion, makes an appeal to popular opinion” Appeal to Emotion An Appeal to Emotion occurs when emotions or emotionally- charged language is used in an attempt to persuade the reader Example: “Officer, please do not give me a ticket for speeding. In the last month I’ve been fired from my job, kicked out of my apartment, and my car broke down. I don’t deserve this! - “attempts to persuade by making an emotional appeal” - “uses emotive language in labelling the proposals” - “the argument appeals to emotion rather than reason” Survey Errors - Biased sample - Improperly constructed/confusing survey questions/ misleading questions (how should the U.S. government withdraw from the United Nations – presumes that the U.S. government should withdraw, a course of action the respondents might not agree with) - Respondents give inaccurate responses - “uses evidence drawn from a small sample that may well be unrepresentative” - “generalizes from an unrepresentative sample” - “states a generalization based on a selection that is not representative of the group about which the generalization is supposed to hold true” Exceptional case/overgeneralizati on This error takes a small number of instances and treats those instances as if they support a broad, sweeping conclusion - “supports a universal claim on the basis of a single example” - “The argument generalizes from too small a sample of cases” - “bases a general claim on a few exceptional instances” Errors of Composition and Division Composition and division errors involve judgments made about groups and parts of a group. An error of composition occurs when the author attributes a characteristic of part of the group to the group as a whole or to each member of the group Example: “Every party I attend is fun and exciting. Therefore, my life is fun and exciting” An error of division occurs when the author attributes a characteristic of the whole (or each member of the whole) to a part of the group Example: “The United States is the wealthiest country in the world. Thus, every American is wealthy.” - “assuming that because something is true of each of the parts of a whole it is true of the whole itself” - “improperly infers that each and every scientist has a certain characteristic from the premise that most scientists have that characteristic” - “takes the view of one lawyer to represent the views of all lawyers” - “presumes, without providing justification, that what is true of a whole must also be true of its constituent parts”
  • 54. COMMON ERRORS OF REASONING – 5 Time Shift Errors The mistake involves assuming that conditions will remain constant over time, and that what was the case in the past will be the case in the present or future Example: “The company has always reimbursed me for meals when I’m on a business trip, so they will certainly reimburse me for meals on this business trip.” - “treats a claim about what is currently the case as if it were a claim about what has been the case for an extended period” - “uncritically draws an inference from what has been true in the past to what will be true in the future” Numbers and Percentage Errors Many errors in this category are committed when an author improperly equates a percentage with a definite quantity, or when an author uses quantity information to make a judgment about the percentage represented by that quantity - “the argument confuses the percentage of the budget spent on a program with the overall amount spent on that program” Errors in the Use of Evidence Lack of evidence for a position is taken to prove that position is false Example: “The White House has failed to offer any evidence that they have reached a trade agreement with China. Therefore, no such agreement has been reached.” In the example above the White House may have valid reasons for withholding information about the trade agreement. The lack of confirming evidence does not undeniably prove that a trade agreement has not been reached - “treats failure to prove a claim as constituting denial of that claim” - “taking a lack of evidence for a claim as evidence undermining that claim” Lack of evidence against a position is taken to prove that position is true This error is the opposite of the previous error. Just because no evidence disproving a position has been introduced does not mean that the position is true Example: “There has been no evidence given against the existence of God, so God must exist.” The lack of evidence against a position does not undeniably prove a position - “treating the failure to establish that a certain claim is false as equivalent to a demonstration that the claim is true” Some evidence against a position is taken to prove that position is false The introduction of evidence against a position only weakens the position; it does not necessarily prove the position false. Example: “Some historians claim that a lengthy drought preceded the fall of the Aztec empire. But we know from Aztec writings that in at least one year during the supposed drought there was minor flooding. Thus, the claim that there was a lengthy drought prior to the fall of the Aztec empire is false.” - “it confuses undermining an argument in support of a given conclusion with showing that the conclusion itself is false” Some evidence for a position is taken to prove that position is true The introduction of evidence for a position only provides support for the position; it does not prove the position to be undeniably true. Example: “We know that the defendant was in the vicinity of the robbery when the robbery occurred. Therefore, the defendant is guilty of the robbery.” As the example proves, partial support for a position does not make the position invincible - “the argument takes evidence showing merely that its conclusion could be true to constitute evidence showing that the conclusion is in fact true”